Talk:ECR
From Wikipedia, the free encyclopedia
1) Which of the following is the easiest change for patients to make?
a) Switch to a low fat diet b) Start an exercise program c) Quit smoking d) Find more time to relax e) None of the above
2) A 40 year old construction worker enters your clinic complaining of foot pain. Upon inspection you observe thin shiny skin over the foot. You also notice decreased foot pulses and a noticeable decrease in the amount of hair compared to the other foot. No edema is present. When the patient stands the foot appears deep red in color, but upon elevation it turns very pale. A small ulcer is also noted on the tip of the big toe. He has not felt ill recently, nor are there any red streaks visible on his person. This patient may be suffering from:
a) chronic arterial insufficiency b) chronic venous insufficiency c) acute lymphangitis d) lead poisoning e) a psychological disorder
3) A 60 year old female presents to the clinic complaining of breathing problems. A high pitched wheeze (audible stridor) is heard upon examination. What is the most likely diagnosis?
a) Tracheal obstruction b) Asthma c) Pulmonary Edema d) Chronic Obstructive Pulmonary Disease e) Upper respiratory infection.
4) A 55 year old male presents to the emergency room with severe chest pain. He has a history of hypertension; bp upon examination is 165/90. When asked to localize the pain the patient pointed to the center of his chest and said that it radiates to his back. He described the pain as having a ripping quality, and it is not relieved by changing positions. What is the most likely diagnosis?
a)Acute myocardial infarction b)Acute pericarditis c)Pulmonary embolism d)Aortic dissection e)Gastroesophageal reflux
5) A 19-year old female patient complains of a headache. She is unable to flex her head forward. She is intolerant of bright light and presents with prostration. You suspect a serious underlying cause for these symptoms. These findings are most suggestive of:
a) Brain tumor b) Glaucoma c) Temporal arteritis d) Meningitis
6) 14 year old male presents to your clinic with lower right quadrant pain since this morning. You perform an abdominal exam and find the patient complains of lower right quadrant pain even upon pressure to the lower left quadrant. The patient also shows a positive referred rebound tenderness. There are normal abdominal sounds, and the patient says there has been no change in diet. Which of the following choices should be considered in this case:
a) appendicitis b) ectopic pregnancy c) gastroenteritis d) intestinal block
7) During a physical examination of a patient, a clinician performs a Straight Leg Raise Test on the individual. What problem does the clinician suspect?
a) Upper Motor Neuron lesion b) Lower Motor Neuron lesion c) Inguinal Hernia d) Herniated Intervertebral Disc e) A burst appendix
8) A 25 year old female comes in for a regular physical exam and upon auscultating her chest you hear something below her left breast. The sound lasts throughout systole and does not change intensity. Her most likely diagnosis is:
a) Mitral Stenosis b) Mitral Regurgitation c) Pulmonic Stenosis d) Pulmonic Regurgitation e) Tricuspid Regurgitation
9) A patient presents to the Emergency Department complaining of a moderate pain (determined using the Visual Analogue Scale) in her right shoulder. According to the World Health Organization's recommendations for pain control, which of the following would be considered appropriate therapy for her pain?
a) Aspirin b) Morphine c) Codeine d) Acetominophen e) no pharmacotherapy suggested
10) During an on-field cutting maneuver, an indoor soccer player's foot is caught in the turf and her knee experiences gross valgus. You are the first to perform an examination of this female athlete, and suspecting the classic triad (injury to ACL, MCL, and meniscus), you perform specific maneuvers to assess the integrity of the knee joint. Which test or groups of tests would most likely elicit abnormal findings due to the suspected injuries in this female athlete?
I. Abduction stress test II. Adduction stress test III. Lachman test IV. Posterior drawer sign V. McMurray test
a. I, III, and V b. II, IV, and V c. II and III d. I and IV e. V only
11) A 25 year old laborer spent the entire weekend smoking pot with his friends. On Monday his family goes to the family doctor worried about his behavior. They describe graphic and threatening writings on the walls. The young man thinks he is God. He walks around the house talking about death.
What is the likely diagnosis, and what should the family doctor do next
a) Glioblastoma. Order an MRI scan. b) Acute drug overdose. Urinary drug screen. c) Acute schizophrenia. Prescribe respiradone d) Acute schizophrenia. Do a complete history and physical examination. e) Bipolar disorder. Do a comprehensive neurological examination.
12) A 32 year old business executive presents with complaints of decreased memory, weight gain, lethargy, constipation and hairloss. On physical examination she has a pulse rate of 55 bpm, cool thickened extremities, diminished reflexes , and a blood pressure of 145/90.
What is the most likely diagnosis?
a) Cushings disease b) Thyrotoxicosis c) Hypothyroidism d) Subacute thyroiditis e) Addisons disease
13) During your Internal Medicine Clerkship, you are asked to perform a Head-to-Toe exam. As you test the patient's six cardinal fields of gaze, you know that in order to perform this test properly, your patient must have integrity in all of the following EXCEPT:
a) CN II b) CN III c) CN IV d) CN V e) CN VI
14) A teenaged patient comes to the clinic with knee pain after a side-long collision with another player during a hockey game. You suspect an MCL injury. To test it, you would:
a) Perform a valgus stress test b) Perform an abduction stress test c) Perform a varus stress test d) Both A and B are correct e) Both B and C are correct
15) A 26 year old male patient comes into the emergency room with "the most painful splitting headache ever" that worsened in a matter of minutes. He also complains of nausea, vomiting, neck pain, and blurred vision. His wife notes that he "seemed confused" during the drive to the hospital. Physical exam findings include a blood pressure of 200/140 (his wife tells you that his blood pressure is normally 120/90), pulse of 44 beats per minute, and an rhythmic yet abnormal respiratory pattern of alternating rapid and absent periods of breathing. You also find a unilaterally enlarged, oval-shaped right pupil that is slow to react to light as well as papilledema on fundoscopic exam. Based on these symptoms, you come up with a diagnosis. What is your immediate next course of action?
a) prescribe a beta blocker because his problem is primarily the high blood pressure and it is way out of normal range b) do a lumbar puncture right away because you suspect he might have meningitis c) wedge something in his mouth and wait because you are absolutely sure he is about to have a seizure d) order an emergent CT because you suspect he has a subarachnoid hemorrhage e) give him morphine for the pain because the source of his problems is really the headache that is causing these other symptoms
16) A patient presents with facial paralysis, sensory loss on half of the face, Horner's syndrome, paresis of conjugate gaze and abduction of eye. What is the likely artery and location affected?
a) Basilar Artery; Cerebrum b) Basilar Artery; Brainstem c) Middle Cerebral Artery; Cerebrum d) Posterior Cerebral Artery; Brainstem e) Anterior Cerebral Artery; Cerebrum
17) Upon doing a physical examination, you notice that when you test the corneal reflex using a wisp of cotton, the patient's eyes do not blink. Of the following, which cranial nerve is possibly involved with such finding?
a) CN III b) CN VII c) CN IV d) CN VI e) None of the above.
18) An unconscious patient is rushed to to the emergency room following a car accident. Upon arrival a resident checks the patient's vital signs including the patient's respiratory rate. The resident finds it difficult to determine the respiratory rate because the patient's respiration is irregular with a random pattern of deep and shallow respirations followed by short periods of apnea. Due to this abnormal respiratory pattern, the resident predicts that the patient has a brainstem lesion at the level of the medulla. What type of abnormal respiratory pattern did the resident observe?
a) Cheyne-Stokes Breathing b) Central Neurogenic Hyperventilation c) Apneustic Breathing d) Cluster Breathing e) Ataxic Respirations
19) A 45-year old woman comes into the clinic complaining of some loss of sensation in her face and scalp and blurry vision. While performing the exam, you discover that she has no corneal reflex on the right side. Sensation testing is normal throughout the entire left side of her face, as well as the lower portion of the right side of her face. Jaw movements and reflexes are also normal on both sides. Damage to which nerve is most likely the cause of this lack of corneal reflex?
a) Right Facial Nerve (CN VII) b) Right Maxillary branch of the Trigeminal Nerve (CN V2) c) Right Ophthalmic branch of the Trigeminal Nerve (CN V1) d) Right Oculomotor Nerve (CN III) e) Optic Nerve (CN II)
20) During a physical exam of a patient you wish to measure their jugular venous pressure. In order to make the measurement the patient should be...?
a) Lying flat on the examination table b) Standing in the upright position c) Elevated to approximately 60 degrees on the examination table d) Lying on their stomach on the examination table
21) Upon auscultating your patient's lungs you detect adventitious sounds, which you determine to be wheezes. Being an excellect clinician you know that wheezes could suggest all of the following, EXCEPT:
a) asthma b) COPD c) bronchitis d) secretions in large airways e) narrowed airways
22) A middle age women presents to your clinic. Which one of the following symptoms (or physical characteristics) does NOT increase her risk of coronary heart disease?
a) African American descent b) Diabetes c) HDL >60 mg/dl d) Chronic Renal Disease e) A body mass index (BMI) >29
23) A 43 year old male was working construction on the hospitals parking garage when he began experiencing a substernal "squeezing" pain that radiated to his left arm. Concerned that it could be a heart attack his friends quickly bring him to the ER. A quick history reveals that he smokes one pack a day for the past 20 years and that he has had similar pain that normally subsides with rest. Being an excellent clinician you give the patient sublingual nitroglycerin and within 5 minutes the pain is relieved. What is the most likely diagnosis?
a) Acute Myocardial Infarction b) Aortic Dissection c) Chronic Stable Angina d) Unstable Angina e) Acute Pericarditis
24) A 23 year old patient presents to the clinic with complaints of chronic, painless diarrhea. When asked about her bowel movements, the patient reveals that in the past 3 months she has experienced changes in the frequency and form of her stool. Which of the following would be the best diagnosis for the patient?
a) Diverticular Disease b) Irritable Bowel Syndrome c) Intestinal Pseudoobstruction d) Ischemic Colitis e) Hemorrhoids
25) A 35 yr old female patient complains of recurrent headaches on one side of her head that sometimes last for three days. She has noted that the onset of the excruciating pain usually occurs before her monthly menstrual cycle. Increased stress and alcohol seem to provoke the headache and loud noises and bright light make it worse. Based on this information which of the following would be the most likely diagnosis?
a) brain tumor b) cluster headache c) aneurysm d) migraine e) tension headache
26) During your first rotation as a third year medical student you perform a basic physical exam on a 68 y.o. gentleman at the VA. You notice that when asked to say "ah," the patient's uvula deviated to his right and when asked to put his tongue out of his mouth the patient's tongue also deviated to his right. Excited, you report these findings to your attending because you know this means a paralysis to:
a) Left cranial nerve X and left cranial nerve XII b) Left cranial nerve X and right cranial nerve XII c) Right cranial nerve X and left cranial nerve XII d) Right cranial nerve X and right cranial nerve XII e) You are not excited because these are normal findings, especially in the elderly.
27) A 23-year old male patient presents with a dull, gnawing pain in his epigastric abdomen. It does not radiate. Food seems to relieve the pain but it comes back after an hour. Pain occurs over the course of a few weeks, disappears inexplicably, then comes back again; this has been going on for over a year. Patient often feels pain during the night making him very uncomfortable and as if he hasn't eaten for a long time. Over the counter antacids have no effect. No other symptoms or history of related disease or procedures. Negative Murphy's sign, no rebound tenderness. Most likely diagnosis?
a) Duodenal ulcer b) Cholelithiasis c) Food poisoning d) Diverticulitis e) Pancreatitis
28) A 47 year old Asian male presents to the clinic with chief complaint of back pain and a slight tingling in his trunk (which started yesterday). Tenderness and pain began in his upper back between his shoulder blades about 4 weeks ago, and the pain is worse when the patient lies down. Which of the following is the best course of action? (Answer listed on bottom, in case you don't want to see it immediately).
a) Emergency decompressive laminectomy b) Lumbar puncture to test CSF c) Symptomatic care; the patient most likely has Herpes Zoster d) Schedule an immediate MRI to test for neoplasm e) Order CT with contrast to rule in diagnosis of herniated disc
29) a 45 year old woman presents to your office complaining of fatigue, weight gain, and hair loss. On examination you observe neck fullness. When you palpate her thyroid, it is enlarged, smooth and nontender. You order thyroid function testing: her T4, free T4, and T3 are normal, but her TSH is slightly elevated. What is the most likely diagnosis:
a) Iron deficiency b) Grave's disease c) Hashimoto thyroiditis d) Thyroid cancer e) Multinodular goiter
30) A 39 year old female comes to ER complaining of cramping in the upper abdomen that "comes and goes." It started three days earlier. She also reports nausea and vomiting of a "greenish substance." Inspection of the abdomen reveals slight distension. Upon auscultation of the abdomen you hear hyperactive borborygmi. Vitals are normal. The patient is most likely suffering from:
a) Salpingitis b) Pancreatitis c) Small bowel obstruction d) Appendicitis e) Large bowel obstruction
31) The Screnning assessment test CAGE is used to assess alcoholism. Which question is NOT one that CAGE ask?
a) Do you ever felt the need to cut back/cut down on your drinking? b) Do you feel annoyed at all by people criticizing your drinking? c) Have you felt guilty at all by your drinking habit? d) Do you feel the urge to share your drink with other people? e) Have you felt the need to have some sort of an 'eye-opener' in the morning?
32) A 22 year old woman comes into your office complaining of recent episodes of terror accompanied by dyspnea, chest pain, dizziness, and faintness. She has experienced such episodes 7 times within the last month, and she feels anxiety in anticipation of another attack. What is the proper diagnosis?
a) Generalized Anxiety Disorder b) Obsessive Compulsive Disorder c) Panic Disorder d) Posttraumatic Stress Disorder e) Schizophrenia
33) A 43 year old woman makes an appointment for a physical after her 49 year old brother had a heart attack. She smokes half a pack of cigarettes a day but does not drink. Her blood pressure is 136/94. There is a family history of type 2 diabetes, and her LDL is 140 mg/dl. What is she most at risk for developing?
a) lung cancer b) coronary heart disease c) asthma d) nephrotic syndrome
34) A worried mother brings in her 16 year-old son for counseling as she has recently learned of his engagement in substance abuse. The patient denies the accusation, but he does admit trouble sleeping at night and thoughts of suicide. When asked about friends, the patient explains that he is not a social person and prefers spending time at home. He also discusses the stress of being the eldest of his siblings as his parents seem to expect too much of him. An odd behavior is noted in the middle of the session, when the patient all of a sudden asks for a brief moment of silence during which he seems to count something with his hands. When the mother is brought back into the room to discuss future visitations, the physician observes some motor and phonic tics. When asked about them, the mother explains that she has been diagnosed with Tourette syndrome. What does the 16 year-old patient likely to have?
a) Schizophrenia b) Social Phobia c) Bipolar Disorder d) Obsessive-Compulsive Disorder e) Panic Disorder
35) A 22 yo male comes to the clinic complaining about recurrent pain on one side of his head for the last 6 weeks that seems to worsen at night. He claims that each night at roughly the same time he wakes up due to the severity of the pain. He also noted that when he drinks it seems to induce these same symptoms. With this information which of the following would be the most likely diagnosis?
a) Stroke b) Tension Headache c) Elevated Cholesterol d) Cluster Headache e) Aneurysm
36) A 28 year-old graduate student, non-smoker, who lives in an urban area, is at the hospital for a checkup. He explains feeling "down" frequently over the course of the past couple of months, with no reason at all. He does not find joy he use to in his hobbies or friends. He also complains of having no energy, appetite or motivation to work. Normal levels of TSH, T4 and T3 were found. What is another possible condition that this patient may exhibit other than depression?
a) Congestive Heart Failure b) Alcohol dependency c) Thyroid Hormone Deficiency d) Pneumothorax e) COPD
37) A 53 year-old man who says he is a guitarist presents himself to your clinic. He has bad "shakes", hypertension, and tachycardia. He is also extremely confused and agitated. His blood test shows that triglycerides level is elevated and he is moderately anemic. Your first choice of treatment for him would be:
a) Benzodiazepine b) Methadone c) Imipramine d) Clozapine e) Radioiodine
38) A 45 year old pregnant woman comes into your office presenting with various symptoms. She presents with a goiter, which is soft upon palpitation and a bruit can be heard with a stethoscope. The patient also complains of an increased appetite and frequent bowel movements after meals. Suspecting a thyroid disorder, you order a serum sample and the lab results reveal low TSH and high free T4. What is the proper diagnosis and proper treatment?
a) Hypothyroidism – treat with 65 ug of levothyroxine daily b) Toxic Multinodular Goiter – treat with antithyroid drugs and follow with surgerical removal of the goiter c) Graves' Disease – treat with antithyroid drugs such as methimazole and follow-up 3-4 weeks after initial treatment d) Sick Euthyroid Syndrome – perform partial thyroidectomy with lymph node dissection e) Graves' Disease – treat with radioiodine
40) A 5-year-old male patient presents to your office with nasal drainage, congestion, facial pain over cheeks, and headache for 10 days. Facial pain is worse when patient bends over. What is the most likely diagnosis?
a) Chronic bacterial sinusitis b) Acute bacterial sinusitis c) Nonspecific URI (Viral) d) Fractured nasal bone e) Chronic fungal sinusitis
41) A 25-year-old patient is complaining of frontal baldness. During your physical examination, you notice that after a hard handgrip she is unable to relax her muscles and appears to have sustained contraction. You also notice that she is intellectually impaired and complains of hypersomnia. You conclude that:
a) This patient might be suffering from Myotonic Dystrophy. b) The patient is missing part of her chrm 4. c) Muscle biopsy shows "ragged red fibbers". d) There could be endocrine abnormalities. e) Both A and D are correct.
42) A 84 year-old male patient was admitted into the ICU a week ago and have been on mechanical ventilation for acute lung injury. His new and recent complains include pleuritic chest pain and dyspnea. His sputum appears blood-streaked. Body temperature is 38 Celsius, and his lab CBC report >10,000 WBCs/µL. A chest x-ray confirms a new infiltrate in his lung, and bronchoscopic examination found P. aeruginosa. What is the new condition that can be suspected to cause the current symptoms?
A. Congestive heart failure B. Hospital-acquired pneumonia C. Lung cancer D. Community-acquired pneumonia E. Pulmonary embolism
43) A young father brings his 2-month-old daughter to the ER. She is pale and restless, and the father states she has been crying for several hours. He states, "I have tried everything, but nothing works! She won't stop crying!" You note the child is cool to the touch, and her skin is clammy. When she cries, you note a slight wheezing sound, and her nostrils are flared. You also observe that the capillary refill time of her toes is increased from normal (it takes longer for color to return). What is the most important feature of the history/physical that will help you determine if the child is truly sick?
A. Child is cold and clammy B. Child is wheezing C. Child has increased capillary refill time D. Child is pale and crying for several hours E. Child demonstrates nasal flaring
44) A hypertensive, 65-year-old man presented to the ER with a sudden, severe, tearing abdominal pain that radiates to his back. The man's vitals reveal that he has hypotension, tachycardia, and tachypnea. During the physical examination, the man is found to have a pulsatile mass in his abdomen. He is also confused, in a delirious state, and has cool and clammy skin. Based on the given information, you conclude that this individual has...
A. Carpal Tunnel Syndrome B. Diverticulosis C. Small Bowel Obstruction D. Pancreatitis E. Abdominal Aortic Aneurysm
45) A 60-year-old male comes into office complaining of problems sleeping. He notes he gets out of breath during exercise. He complains of tightness in his chest, but no pain. He has not been ill recently, nor had previous symptoms. Upon examination, he has an irregular heart rate and auscultation produces crackles. What is most likely this man's problem?
A) Pneumonia B) Airway obstruction C) Left Heart Failure (I think) D) Right Heart Failure E) Asthma
-
- Guys, I tried looking up the answer, but I don’t know for sure…
46) A new patient presents to your office with a chief complaint of the inability to sleep at night. While taking the HPI, the patient reveals that they have recently felt that something is "just not right" in her life. It seems that something bad is bound to happen to her at any moment it has begun to interfere with her ability to concentrate and be productive at work. She denies any recent changes or major events in her life (marriage, death, divorce, etc) and does not have any hypothesis for the cause of their symptoms. Upon physical examination, her muscles seem abnormally tense, yet respiration and heart rate fall within the normal range. Based on these symptoms, what is the most appropriate next step?
A) Advise the patient to take time off work to relieve some stress B) Prescribe anti-histamine to help her sleep C) Prescribe a benzodiazepine to relieve generalized anxiety D) Order a CT to rule out cerebral lesions or tumors that might be causing her psychological disturbances E) Prescribe a muscle relaxant; it should relieve the tenseness and probably help her sleep too
47) A 67-year-old woman comes into the office with complaint of pain in the joints of her right hand. She says the pain is worse in the morning and after heavy use of that hand, and sometimes she finds it difficult to bend her fingers at the knuckles. Upon physical examination, you find the presence of Heberden and Bouchard's nodes, and joint crepitation. There was otherwise no tenderness or inflammation in her hand. She is in good general health, and her complete blood count came back normal. This patient most likely suffers from:
A) Rheumatoid arthritis B) Osteoarthritis C) Gout D) Ankylosing spondylitis E) Nothing; these findings are normal for a woman her age.
48) A 16-year-old female presents in the emergency room with a chief complaint of a throbbing headache. The pain is unilateral and has not improved in the last four hours. She is nauseous and tells you that the light is making her pain worse. The patient notes that she has had headaches before however she is scared because before her current headache began she saw flickering zigzagging lines. Upon questioning the patient, you learn that she is premenstrual. What is the most likely diagnosis?
A) Cluster headache B) Tension headache C) Meningitis D) Migraine headache E) Sinus headache
49) A 17-year-old patient comes to your office complaining of having noticed a dramatic change in behavior as of late. She says that she notices certain habits that she cannot resist the urge to perform and it is disrupting her daily lifestyle if they are not done properly. The best way describe this patient's tendencies would be:
A) Anxieties B) Phobias C) Obsessions D) Compulsions E) Perseveration
50) A 58-year-old man presents to the ER with severe chest pain across his anterior chest that is radiating to his neck and back. He describes his pain as ripping or tearing, and changing positions does not alleviate it. Upon checking his pulses, you notice that his carotid pulses are asymmetric. This patient most likely has:
A) Pericarditis B) Myocardial Infarction C) Reflex Esophagitis D) Dissecting Aortic Aneurysm E) Angina Pectoris
51) A 95 y/o overweight patient who has had type 2 diabetes for over 50 years is brought to you in a wheel chair. Physical exam shows paralysis of his left foot and leg. Furthermore, the patient complains that he cannot hold his urine anymore. Which of the following is most likely the cause?
A) Occlusion of right anterior cerebral artery B) Occlusion of left anterior cerebral artery C) Occlusion of right middle cerebral artery D) Occlusion of left middle cerebral artery E) All of the above
52) An 80-year-old female patient comes in complaining of hearing loss in her right ear. Oddly enough, she also mentions that she can hear better when her daughter talks to her in a busy mall. You suspect that her right ear might have some wax accumulation but your otoscope is broken. What test that you can you perform to confirm this theory without visual confirmation and what would be the clinical findings?
A) Rinne Test: bilateral conductive hearing loss B) Weber's Test: bilateral sensorineural hearing loss C) Weber's Test: unilateral sensorineural hearing loss with sound heard better in the left ear D) Rinne Test: sound in the right ear is heard through bone longer than it is through air E) Weber's test: unilateral conductive hearing loss with sound heard better in the right ear
53) You are evaluating a 45-year-old male with a chief complaint of abdominal pain. Which of the following signs/symptoms/history elements would NOT indicate a diagnosis of pancreatitis? A. A history of alcohol abuse or alcoholism B. Elevated Serum Amylase C. The patient complains of pain radiating to right shoulder D. The patient says his pain is located in the epigastric region E. The patient complains of fatty, foul-smelling stool (steatorrhea)
54) A 32-year-old female comes in complaining of fatigue, loss of appetite and weight gain. Upon examination, she is found to have cold hands and feet, bradycardia, mild diastolic hypertension and a prolonged relaxation phase of deep tendon reflexes. Blood tests show decreased levels of serum T4 and elevated TSH, in addition to a positive thyroid peroxidase antibody test. The most likely diagnosis is:
A. Autoimmune hypothyroidism B. Iodine deficiency C. Grave's disease D. Sheehan's syndrome E. Toxic multinodular goiter
55) A 50-year-old male patient comes in complaining of a pain in his right shoulder. He explains that the pain started to occur right after one throw of a football to his grandson while playing catch. One possibility you suspect may be a tear in the supraspinatus. Which of the following results will least likely support his claim?
A. The patient is unable to hold the arm fully abducted at shoulder level. B. The patient experiences pain when asked to adduct his arm over his chest. C. The patient has trouble reaching his left scapula with his right hand. D. The patient is unable to raise the arm above shoulder level. E. You feel tenderness when palpating the muscles that insert on the greater tuberosity of the humerus.
56) A 65-year old woman presents with a chief complaint of leg and back pain. History and examination reveal that the pain is bilateral when the patient is standing and walking, but the patient says that it does not hurt when she is sitting down. Sensory examination reveals that the patient lacks sensation in both feet. What is your diagnosis?
A) Radiculopathy at L5/S1 B) Neoplasm of spine C) Lumbar disk disease at L5/S1 level D) Spinal Stenosis E) Vascular claudication
57) While you were interviewing a patient, you noticed the following: the patient's speech was characterized by indirection and delay in reaching the point because of unnecessary detail, although components of the description have a meaningful connection. Please choose the best answer that describes this finding in the patients' thought process.
(A) Flight of ideals (B) Loose association (C) Circumstantiality (D) Neologisms (E) Echolalia
58) A 25 year-old male comes to the Emergency Department complaining of periumbilical pain, which started out mild but has gotten steadily worse over the past 4 hours. The man states that his pain worsens when he moves or coughs. He has no desire to eat anything due to his nausea, and a physical exam reveals a low-grade fever. The most likely diagnosis is:
A) Peptic Ulcer B) Biliary Colic C) Chronic Pancreatitis D) Acute Appendicitis E) Acute Mechanical Intestinal Obstruction
59) You encounter a middle-aged man on rounds complaining of anxiety, tremor, blurred vision and trouble sleeping. He says that he was admitted to the hospital 12 hours ago after he was found sleeping on a park bench by the police. You notice that he has stitches on his scalp. The man is most likely experiencing
A. A seizure B. Sinusitis C. A stroke D. Alcohol withdrawal E. Schizophrenia
60) A patient presents with a history of 2-4 drinks of alcohol per day. Which of the following questions would NOT be included to assess the level of the patient’s alcohol abuse/dependence?
A) Have you ever felt guilty about your drinking? B) Have you felt annoyed with questions of your drinking? C) Have you ever felt the need for an eye-opener in the morning? D) Have you ever passed out drunk before? E) Have you ever felt the need to cut down on your drinking?
61) Which of the following illnesses is NOT typically associated with an increased risk for depression?
A. Parkinson's Disease B. Pneumonia C. Angelman Syndrome D. Stroke E. Myocardial Infarction
Module 9: March 7th Powerpoint Slides 17-22. Answer could also be Pneumonia because it is not listed under the slide heading entitled “Medical Disorders associated with an increased risk for Depression”. Instead, it is listed under the next heading of “Medical conditions that are significantly affected by depression”. Angelman Syndrome, a neuro-genetic disorder, is however not listed anywhere between slides 17-22.
62) A 45-year-old man reports to the ER with the following complaints – nausea, vomiting, and persistent epigastric pain that radiates to his back. He also prefers to sit leaning forward with his trunk flexed. The patient also reports of having similar episodes in the past and confides to be an alcoholic. While performing a physical examination, you notice presence of epigastric tenderness, and negative Murphy’s test. Which of the following diagnosis fit appropriately with the patient's symptoms?
a. Biliary colic b. Acute pancreatitis c. Acute cholecystitis d. Acute diverticulitis e. Acute appendicitis
63) A 61-year-old female pt presents with poor hearing in the right ear. Upon performing the Rinne test, she can hear the sound through the mastoid bone longer than through air. What kind of hearing loss the patient has?
a) Sensory neural hearing loss b) Conductive hearing loss c) Visceral hearing loss d) Vestibular Cochlear nerve hearing loss e) Bitemporal Hemianopsia.
64) A 55-year-old man comes into your office for a routine checkup. Upon examination, you discover a heart murmur that was not present when you last examined him. After taking a more in-depth history, you discover that he had rheumatic fever as a child -- a complication of strep throat. Being an excellent clinician, you suspect a problem with the mitral valve because it is commonly damaged by rheumatic fever. Where is the best position to place your stethoscope to hear the mitral valve most clearly?
A: 2nd intercostal space along the right sternal border B: 2nd intercostal space along the left sternal border C: 3rd to 5th intercostal space along the left sternal border D: 4th or 5th intercostal space along the left mid-clavicular line E: 5th or 6th intercostal space along the left mid-clavicular line
65) An 80 yr. old women is brought into your office by her middle-aged daughter. The daughter explains that her mother has difficulty getting around and seems to becoming more forgetful. She is concerned about her mother’s health and thinks her mother should not be living alone. You decided to perform a functional assessment. Which activity would you ask about first? A) Dressing B) Bathing C) Eating D) Shopping E) Going to the bathroom
66) A 54-year-old female presents with a "severe headache" that began 1-hour prior. The patient reports nausea, vomiting and neck pain, but denies fever. The patient also reports experiencing visual changes. Not far into the HPI, the patient vomits again and loses consciousness. The most likely diagnosis is:
A. Subarachnoid hemorrhage B. Bacterial Meningitis C. Migraine headache D. Brain Tumor E. Sinusitis
67) A 54 y/o male comes to the emergency complaining of coughing and a burning pain in his chest. He points to the upper part of his chest on either side of his sternum. Coughing makes the pain worse and lying on one side seems to relieve it. Upon inspection, it was found that the patient is suffering from viral infection and has an inflammation in his upper respiratory system, specifically his trachea and large bronchi. Which of the following diagnoses is likely?
A. Bronchiectasis B. Pleural Pain C. Tracheobronchitis D. Pneumonia E. Chronic Bronchitis
68) A 45 yo male patient presents with extreme tenderness in the metatarsophalangeal joint of his right big toe for the past couple of days. His big toe is inflamed, hot and red and he presents with a minor fever. Patient disclosed that the pain began two nights prior after a night of drinking. The patient denies stubbing or injuring his toe. You collect fluid from the joint and find crystals, which confirms your suspicion of: A) Rheumatoid Arthritis B) Osteoarthritis C) Acute Gout D) A stress fracture E) Cellulitis
69) A 17-year-old male patient presents with knee pain and swelling. When asked to describe the onset of the pain, the patient explained that he heard a pop and felt a lot of pain before his knee instantly swelled up. Which imaging modality is best suited to verify the most likely diagnosis for this patient's knee pain?
A) CT Scan B) Ultrasound C) MRI D) X-ray E) Angiography
70) A male patient (57 yrs.) with a history of hypertension and smoking presents at the ER with miosis, ptosis, and anhydrosis of the left eye/face. Funduscopic exam is normal. What is the most likely explanation for these sets of symptoms? A. Tumor, that has obviously interrupted the parasympathetics to the left eye B. Amaurosis fugax C. He probably has uncontrolled diabetes and is suffering from peripheral neuropathy in the eye innervations D. Damage to the Sympathetics, most likely from stoke in the vertebral artery feeding the lateral medulla E. Bell's Palsy is the most likely diagnosis due to the history of smoking and hypertension
71) A 27 year old white male who is six foot three, weighs 255 pounds, and who is in near peak atheletic conditioning appears at your clinic. As a good physician you start by taking vital signs, you become very stumped when you find the patients blood pressure to be 250/100. What conditions should you check in order to make sure you have taken an acurate reading?
A. Make sure the pressure cuff is not too short B. Make sure the pressure cuff is not too narrow C. Make sure the brachial artery is at heart level D. Make sure the cuff's bladder does not ballon outside of the cuff E. All of the above
72) A 35 year old female presents to your office complaining of severe pain radiating to the groin, accompanied by nausea, vomiting, and light-headedness. She states that she noticed blood in her urine, and in her chart you find a history of frequently recurrent UTI. Urine culture shows presence of Proteus and Klebsiella organisms. Which one of the following choices will NOT contribute to this patient's treatment plan?
A) Increase fluid intake B) Decrease dietary calcium C) Moderate protein intake D) Suppressive antibiotic therapy E) Moderate sodium intake.
73) Patient X comes into the your office complaining that the rings on her left hand and the watch on her left wrist have been abnormally tight for at least a week. She also wears rings on her right hand and has no noticeable signs on that hand, nor does she have any visible changes in size of the right arm/hand. This morning the patient awoke to find that ONLY her left upper extremity (arm, forearm, hand) were so enlarged that she could hardly fit them through her shirt sleeve. After taking the patient's history, which piece of the history is MOST likely to attribute to the symptoms she is describing.
A) The patient has numerous family members who have suffered from right heart failure. B) The patient was recently in a car accident and in the ER an x-ray was taken that showed a fracture in the T4 vertebrae. No other injuries were recorded. C) The patient has recently been prescribed a dosage of Lasix (a loop diuretic) by another physician and she takes the drug regularly. D) The patient has been a smoker for 20 years and only feels comfortable sleeping with at least 2 pillows to raise and support their head. E) The patient was diagnosed one month ago with breast cancer of the left breast and infiltration into her left axillary lymph nodes. She underwent surgery to remove the affected nodes and breast cancer two weeks ago.
74) A patient presents in your office complaining of difficulty hearing lately. You did a Weber test on the patient with a 512 Hz tuning fork. After executing the test you found out that the patient could hear the sound much better on the left ear when the tuning fork is placed on top in the middle of the patient's head. You conclude that the patient has:
A) Unilateral conductive hearing loss of the right ear. B) Unilateral conductive hearing loss of the left ear. C) Unilateral sensorineural hearing loss of the right ear. D) Unilateral sensorineural hearing loss of the left ear. E) The patient's hearing is fine
75) A 20 year old presents with fever, headache and a stiff-neck. If upon flexion of the neck, there is flexion of hips and knees, the patient is said to have a positive a)Brudzinski's sign b)Kernig's sign c)Lachman's sign d)Anterior drawer sign
76) As a third year medical student doing your surgical rotation, you just saw your 35 year old female patient that had an appendectomy two days ago. You wrote an order for her to get out of bed and to ambulate three times a day. An hour later, you received a page from your attending that the patient is complaining of substernal pain, pleuritic in nature. She had several episodes of hemoptysis and heart rate is in the 140's.The arterial blood gas reveals severe hypoxemia. What is the most likely diagnosis?
A- Acute Pericarditis B- Pleurisy C- Aortic dissection D- Pulmonary embolism E- mediastinal emphysema
77) A 25 year old female presents to your emergency room with the chief complaint of head ache. She describes the pain as throbbing and can't really localize it to a specific area. She says that the head ache came on rapidly, which is why she was concerned enough to come in to the ER to be seen. During your examination you note that her temperature is elevated and her neck is resistant to passive flexion. The most definitive diagnostic test which would confirm what you believe to be the source of her head ache would be:
A. head CT B. MRI C. plain radiograph D. lumbar puncture E. ask her the CAGE questionnaire to evaluate her for possible alcohol abuse
78) A 34 y/o female patient complains of "tingling and electric sensations" and numbness in the 2nd finger of her left hand since 2 weeks ago. Now the symptoms has progessed to the 3rd finger. And you noticed that she is 6 months pregnant. What are some of the results of specific tests or inspection that you will most likely see in this case?
A) Corn appeared most obviously on 2nd finger B) A negative Anterior Drawer Sign and positive McMurray test C) Bouchard's node at the distal interphalangeal joints of 2nd and 3rd finger D) Thickened plaque overlying the tendons of 2nd and 3rd finger E) A positive Tinel's sign and Phalen's test.
79) A 25 year old, male, medical student complains of "feeling weak and down." After a thorough history, you find that he has been having frequent diarrhea for the past year, usually painless but sometimes accompanies abdominal pain. He has irregular bowel habits. Pain is relieved by defecation. He said his stools had been loose, pasty or thin, and sometimes mucus is present. He denies of blood in stool. What is the next step?
a). Sigmoidoscopy to rule out colon malignancy and inflammation. b). Refer the patient to a psychologist department. c). Colonoscopic laser coagulation. d). IV fluid and antibiotics. e). Prescribe antidepressants.
80) A patient comes to see you in your office. She is complaining of cold intolerance, dry skin, and lethargy. You notice that her skin is very pale and she has periorbital puffiness. You automatically suspect hypothyroidism and order blood work. Which marker is sensitive for distinguishing primary from secondary hypothyroidism?
a. T3 b. T4 c. TSH d. Presence of goiter e. TBG
81) A patient comes into your office complaining of fatigue and difficulty concentrating. Upon further questioning you learn that she has decreased appetite and sleeps a lot during the day. You suspect that the patient could have major depression but have not ruled out the possibility of hypothyroidism. Which of the following pieces of information would NOT be supportive of a diagnosis of major depression?
a) The patient had increased feelings of guilt b) The patient had recent thoughts of suicide c) The patient does not have cold intolerance d) The patient has a lower than normal heart rate e) The patient multiple somatic complaints that are widespread
82) A 45 year old man comes into your clinic and complains about lost of breath during exercise. He has also been experiencing fatigue. Upon physical examination, you found his heart rate to be 100 beats per min. His hands and gums appear relatively pale compared to the normal appearance. What is the most likely diagnosis?
A) Infraction in the brainstem B) Moderate anemia C) Hyperthyroidism D) Acute pericarditis E) Pulmonary edema
83) A 58 year old female patient presents to a podiatric medical clinic with a history of pain in her right ankle when she walks. The pain began when she stepped in a hole yesterday while gardening. You suspect a lateral ankle ligament sprain. Which of the following is MOST likely to be true for this patient.
A. The patient has pain on eversion of the foot and little to no pain on inversion of the foot. B. The patient has pain on inversion of the foot and little to no pain on eversion of the foot. C. The problem is most likely due to a subluxation of the first metatarsophalangeal joint. D. The deltoid ligament is most likely the sprained ligaments in this patient. E. The posterior talofibular ligament is more likely to be affected than the anterior talofibular ligament.
84) A patient presents to the ED with severe abdominal pain. The patient is doubled over in pain and groans when you help him sit up. You perform a complete abdominal exam and hear no bowel sounds. Which of these potentially life threatening inflammatory diseases should be treated non-surgically? A. appendicitis B. incarcerated hernia C. infracted bowel D. cholycystitis E. PID
85) A 72-year-old male presents to your primary care practice complaining of a cough that has become progressively worse over the past week. He currently has a mild fever of 38.2 degrees Celsius. He denies having other symptoms besides a mild headache, which he says he suffers from frequently and doesn't think is significant. Upon examination of his thorax, you notice that the percussion note in the lower portion of his right thorax is dull compared to the same level on the left side. Also in this lower right region you notice bronchial - rather than normal vesicular - breath sounds. The patient displays marked egophany and whispered pectoriloquy over this area as well. What would be the next logical step in the management of this patient?
A. Intubate the patient B. Measure the patient's ventilatory function, taking special note of his FEV1 value C. Treat the patient's symptoms with pain medication, rest, and hydration. D. Obtain AP and lateral chest x-rays. E. Obtain a complete blood count, looking for eosinophilia.
86) During your clinical rotations you interview a 37 year old male patient who is complaining of abdominal pain. He describes it as a recurrent left lower quadrant pain relieved by defecation, with alternating constipation and diarrhea.
The lower left quadrant location signals you to consider a diverticular disease, which is herniations of the mucosa through the muscularis. However, you want to be sure and rule out Irritable Bowel Syndrome, the most common GI disease in clinical practice. For this you would recommend which lab or radiologic study?
A. mesenteric arteriography B. serum glucose C. surgery D. ultrasonography E. barium enema
87) During your morning routine, you like to review each of your patient's charts for the day. With your luck, you clumsily spill coffee onto the chart of a 50 y/o female patient, obscuring information including a recent diagnosis of a significant disorder. Embarrassed about your mishap, you scour the remaining distinguishable information of the chart for clues to resolve the identity of the final diagnosis. It seems she came in to the office last week complaining of recent weight gain despite the fact she had little appetite. Physical exam revealed bradycardia (55 bpm) and cool peripheral extremities despite a warm ambient room temperature. Tests run revealed increased TPO antibodies, TSH levels, and CPK levels. You couldn't make out the results of the measurements of the CBC, serum thyroxine (T4), or electrolytes. Which of the following choices was most likely the correct diagnosis of the patient at hand?
A. Primary adrenal insufficiency B. Primary hypothyroidism C. Graves' disease D. Cushing's syndrome E. Major depression
88) A 50 year old woman comes into the ER with the chief complaint of a sudden onset of persistent abdominal pain in her epigastric region, with pain radiating to her back. She notes that the pain worsens when she lays down on her back to go to sleep and is accompanied with nausea and vomiting. The patient also notes a history of alcohol abuse. Given this information, what is the most likely diagnosis?
A. peptic ulcer B. mesenteric Ischemia C. acute diverticulitis D. acute pancreatitis E. cancer of the stomach
89) Patient is a 57-year-old man presents with right arm paralysis, left gaze preference with eyes deviated to left, and difficulty finding words to express himself, though comprehension remains largely intact. Upon questioning his sister, who brought him in, you find that this symptoms occurred roughly twenty minutes ago. Further questioning reveals that their father died of "heart disease," and that the patient smokes regularly, though the sister cannot recall how much he does smoke. You send for exams, but are thinking of treatment modalities. With the information given thus far, you are thinking of:
(a) immediate empirical antimicrobials until labs return (b) immediate tissue plasminogen activator (c) immediate aspirin (d) immediate dexamethasone (e) wait until labs return
90) 70 year-old male presents with fever, productive cough, headache, fatigue, and confusion. Upon physical examination, you find tachypnea, dullness to percussion, crackles and a respiratory rate of greater than 30/min. Which of the following tests is NOT useful for establising a diagnosis?
A. Chest X-Ray B. Urine antigen tests C. Sputum stains and culture D. Pulmonary function tests E. Blood cultures
91) A mother brings in her 2 year old child who is having trouble breathing. The child, who is in his mothers arms, appears normal but you notice its takes some work for him to breathe. Based on these two observations what could you say the infant might have?
a) Respiratory Failure b) Shock c) Respiratory Distress d) Poor Circulation e) Further observations are needed
92) A 18 year old female presents to your office with a chief complaint of an earache in her right ear for the past 3 days. Upon examination you note a scaly dermatitis along the lower border of her right ear. Which of the following would NOT help to rule in a diagnosis of otitis externa.
A.) The patient complains of pain when her ear is pulled up in order to insert an otoscope. B.) Upon examination, the eardrum appears to be bulging and red. C.) Moistness in the ear canal. D.) Tenderness in the ear canal. E.) While the eardrum appears normal, the ear canal appears reddened and the patient complains that it is very itchy.
93) A woman who is overweight, in her forties, and has 4 children comes into your office complaining of gas, nausea and vomiting. She experiences right upper quadrant pain that radiates to her back about a half hour after eating. Your most likely diagnosis would be:
a) cholelithiasis b) Zollinger-Ellison Syndrome c) peptic ulcer d) pancreatitis e) appendicitis
94) A 35 year old female presents to your office with complaints of fatigue, loss of stamina, breathlessness, and tachycardia. To assess her, you order a complete blood count (CBC) and notice that her hematocrit levels are 30%, and blood Hemoglobin (Hb) of 95 g/L. What would be the most likely diagnosis, given this information?
A. Chronic Angina B. Anemia C. Hepatitis D. Congestive Heart Failure E. Diabetes Mellitus
95) A 48-year old female is brought to the Emergency Department after a failed suicide attempt. Her ex-husband comes in to see her, mentioning that she is a retired lawyer, an alcoholic, and has Lupus. Which of the aforementioned characteristics does NOT coincide with the main risk factors for suicide?
a) Age b) Alcohol dependence c) Divorced d) Retired e) Female
96) A 57-year-old male smoker presents to his internist complaing of shortness of breath. The diagnosis of COPD is correctly deduced based on all the following signs and symptoms except:
A) Initial presentation with patient leaning forward with lips pursed during exhalation and arms supported on knees or exam table B) Abnormal retraction of lower interspaces of the chest during inspiration C) Wheezing upon ausculation D) Dullness over both lung fields upon percussion E) Decreased fremitus
97) A 54 year old slightly obese male patient is escorted into your emergency by his wife. As you are speaking with his wife you notice that the patient's eye is slightly deviated to a down and out position as he appears to be dozing off, then waking suddenly. His wife tells you that he was complaining about having vision problems. He complained of not being able to look up and had difficulty fixating on an object. She immediately thought something was but her husband aggressively denied any problems but surmised that he was just tired. Instinctively, you believe his condition to be a stroke but what vessel is occluded and hemisphere affected?
A. MCA, Lat. Cerebral Hemi B. PCA, Midbrain C. ACA, Medial Cerebral Hemi D. PCA, Post. Cerebral Hemi E. Basilar, Medulla
98) A patient comes to your practice who complains of headaches. You notice that ever couple of minutes the patient cracks his knuckles exactly twice for each finger and two times per hand. After the patient does this a few times, you ask him to see how long he can go without cracking his knuckles. He lasts several minutes until he becomes extremely uncomfortable and finally becomes extremely anxious until he breaks and cracks his knuckles. All anxiety is then relieved. The patient suffers from: A) Delusions B) Compulsion C) Obsession D) Addiction E) Phobia
99) A 69 year old woman presents to the emergency department at Great Big Center City Hospital. The attending examines the woman and believes that she is suffering an *ischemic* stroke. Which of the following pieces of information was most helpful to the attending in this diagnosis.
A) Symptoms evolved slowly and the patient became drowsy. B) The patient is in severe pain and is aware that something is very wrong with her. C) Neurologic deficits were focal and onset was abrupt and dramatic. D) Neurologic deficits evolved relentlessly over 20 minutes. E) The patient vomited as symptoms progressed.
100) A patient presents with cold intolerance, loose fitting clothes, and goiter. Which diagnostic tests would confirm your hypothesis of Graves disease?
A. Decreased TSH, increased free T4 B. Increased TSH, decreased free T4 C. Low TBO antibodies, low T3 and T4 D. High TBO antibodies, low T3 and T4 E. Increased TSH, increased free T4
101) Dramatic, Emotional, Erratic behavior is found in: (a) Cluster A (b) Cluster B (c) Cluster Z-aplha (d) Cluster A-T1 (e) Cluster CD
102) Which of the following are the categories evaluated in the Glasgow Coma Scale (GCS)?
A) awareness, level of consciousness,coordination B) alertness, reflexes, flexibility C) eye opening, best motor response, best verbal response D) breathing, vision, blood pressure E) heart rate, reflexes, coordination
103) While working your shift in the ER, a forty year old, noticeably overweight women presents to you stating she began feeling sick and having abdominal pain shortly after she ate at White Castle's. The patient points to the upper right region of her abdomen as the area bothering her. During examination, the patient screams at you when you palpate the RUQ while she takes a deep breath. The rest of the exam produced no other remarkable findings. Which of these diagnostic tools would best identify the patient's illness? A) amylase/lipase B) hCG C) ultrasound D) chest xray E) endoscopy
104) A 49 yr. old female presents to your office with a 1 and a half inch circumfrence ulcer located on the proximal medial side of her right ankle. She also complains of chronic diffuse aching of the legs, aggravated by prolonged standing, and edema. Pulses are normal, color is normal or possible cyanotic with petechiae and then brown pigmentation, normal temperature, no gangrene, elevation reduces edema. What is the underlying cause?
1) artherosclerosis 2) deep venous thrombosis 3) chronic venous insufficiency 4) Lymphangitis 5) arterial insufficiency
105) Which of the following is not a cause of increased intracranial pressure?
A) Meningitis B) Hydrocephalus C) Brain Tumor D) Migraine Headache E) Brain Abscess
106) A 50-year old diabetic man presents to your office with chest pain. The episode occurred when he and his wife were on a hiking through the Grand Canyon during their vacation over the past week. He tells you that the pain only occurred after he had been hiking for a while, and it was relieved when he stopped and rested. When you ask your patient to describe the pain, he describes it as squeezing and radiating to his left arm. Your patient says that he is a smoker, but he has not been smoking as much in recent years. He reveals that his father was diagnosed with coronary artery disease at age 48. Upon physical exam, you find that his blood pressure is 143/90. You note an S4 sound while listening with the bell of your stethoscope.
Based on the presented information, your most likely diagnosis of this patient would be:
A. Aortic Dissection B. Non-ST-Elevation Myocardial Infarction C. Acute Myocardial Infarction D. Chronic Stable Angina E. Unstable Angina
107) A 45 yr old male presents to the ER with the complaint of pain in his legs. He first noticed this pain while riding his bike with his son along a 2 mile bike path earlier that afternoon. He noted that even after he sat down on a park bench for about 10-15 minutes the pain did not go away. He claims that the only way he can relieve his pain is by leaning forward. The man also said that while walking the quarter mile back to his home the pain quickly came back and was once again relieved only by him leaning forward in an awkward position. What is the most likely cause of the man's pain?
A. Intermittent claudication B. Raynaud's phenomenon C. Venous stasis ulcers D. Spinal stenosis E. Restless leg syndrome
108) Your patient complains a tremor during purposeful movement. In physical exam You ask the patient to stand with feet together and eyes open. If the patient sway back and forth at this moment what is the most probable clinical condition of this patient? a. Positive Romberg Test b. Proprioceptive defect c. Vestibular defect d. Both proprioceptive and vestibular defect e. A pure cerebellar defect
109) A 17 year old male presents to your office this morning with the complaint of right knee pain. His knee started hurting after he was hit in last night's big football game. At the time of the injury he described hearing a "pop" and felt as if his knee "gave out." This morning the patient is experiencing pain and the knee is visibly swollen. Based on this information you suspect a possible ACL tear. What would be the best technique to test for a possible ACL tear?
A. Anterior Drawer Test B. Posterior Drawer Test C. Straight Leg Raise test D. McMurray Test E. Hawkins Test
110) A 35 year old man is rushed into the E.R. after a serious car accident. You can see there was physical trauma to his head and order a C.T. scan. On the scan, you note that there is blood collected between the dura mater and the skull. You determine this to be an Epidural Hematoma. What vessel is most likely involved with this injury?
a. Internal carotid b. Dural venous sinuses c. Middle meningeal artery d. Anterior meningeal artery e. Bridging veins
111) A 46 year old man presents to his family physician with complaints of pain and extreme tenderness at the base of his right hallux (first MTP joint). The patient relates that he was suddenly awoken by the pain in the middle of the night after an evening of drinking with his buddies. The pain, which the patient described as hot and tender, lasted for 2 days before gradually subsiding. This is the second time the patient has experienced this type of pain, he does not recall injuring his toe, and is concerned that it might be something serious. Based on the patient's history alone, what would your diagnosis be?
a) Systemic Lupus b) Rheumatoid Arthritis c) Acute Gout d) Osteoarthritis e) Fibromyalgia Syndrome
112) A 37 year old female presents to the emergency room with the complaint of abdominal pain. The patient describes the pain as being a steady pain that radiates through to her back. The patient also complains of diarrhea with no nausea. The patient states that the pain began after she took a wine tasting tour with her husband. The patient has had similar pain before that seemed to be triggered by greasy foods. Occasionally, leaning forward can alleviate the pain; however, the pain returns when the patient resumes normal activities.
A. Acute pancreatitis B. Chronic pancreatitis C. Acute cholecystitis D. Cancer of the stomach E. Peptic ulcer
113) A 45 yr. old male is brought into the emergency room by his wife. His wife claims that she is afraid her husband is going to hurt her. The patient lost his job 2 weeks ago. The patient appears uncooperative. Once restrained you discover the patient has a blood alcohol level of .12. The patient also appears to have a bruise on his forehead. The wife claims that the patient thought he was a "bird" and he wanted to see what if felt like to die by flying into a window. In addition, the wife says she stopped her husband from attempting to parachute off their 3rd floor balcony with a sheet. Upon recieving this information you come up with a differential diagnosis of?
A. Alcoholism B. Suicide C. Lesion in the Limbic lobe D. Stage 1 ICP E. Brain Tumor
114) A 24 year old male comes to your office with a headache. He complains of a severe, deep pain around his left eye. He's had it for about 2 hours, but complains he gets several each day and it lasts for weeks at a time. He also has signs of lacrimation, rhinorrhea, eyelid edema, and conjunctival infection. What type of headache is this?
A. Tension headache B. Migraine headache C. Cluster headache D. Headache from eye disorder E. Headache due to meningitis
115) A 52 Year old man has recently checked in to the hospital with a low-grade fever, tachycardia, hypotension, abdominal tenderness, and Cullen's sign. You suspect acute pancreatitis, but need a solid confirmation. Which of these tasks will be least likely to help with the diagnosis?
A. Find if the patient has or has ever had cholelithiasis. B. Conduct an abdominal radiograph. C. Ask about the patient's alcohol consumption. D. Perform an ultrasound on the abdominal region. E. Conduct a CT scan.
116) A 25 yr old man presents in the emergency room with nausea, vomiting and midepigastric pain. He tells you that the pain often increases when he lies and down and sometimes it radiates to the back. You perform a physical exam and lab tests and notice the following: a positive Turner's and Cullen's sign, elevated serum amylase, abdominal rigidity and basilar rales and pelural effusion on the left lung. What disease is your patient most likely to have? a) Peptic ulcer b) Cholecystitis c) Pancreatitis d) Secondary ciliary cirrhosis e) Obstructive jaundice
117) A 20 year old male patient presents to your clinic with the complaint of a headache. He denies regular headaches and ever having this type of headache before and states, "This is the worst pain ever." He also complains of extreme exhaustion that began this morning. Upon examination the patient is sensitive to light and complains of a stiff neck during muscle testing. What is the most likely cause for this patients symptoms?
A) Migraine B) Cluster Headache C) Intracranial Hemmorrhage D) Meningitis E) Brain Tumor
118) A 45 year old man presents to the ER with generalized periumbilical pain in his abdomen which is now confined to the right lower quadrant. The pain was initially mild and then increased to a steady and more intense pain. The patient says coughing and movement increases the abdominal pain and that he experiences nausea and anorexia. Upon examination, the attending finds that pressing on the left side of the patient's abdomen causes pain in the patient's right lower quadrant (positive Rosving's sign). What is his diagnosis? A. Stomach cancer B. Peptic ulcer C. Appendicitis D. Acute diverticulitis E. Biliary colic
119) A 15 year old male presents to the emergency room limping with a swollen right knee. The patient reveals that he was at football practice and twisted his knee and felt it "pop." He relates that it is very painful to walk on. He is otherwise in good health and has never had any previous serious injuries. Upon performing the Lachman test on the right knee, the patient screams in pain as you feel the tibia slip slightly forward. There is no forward movement of the tibia felt when this maneuver is performed on the left knee. What is the most likely diagnosis?
A. torn posterior cruciate ligament B. torn anterior cruciate ligament C. torn medial meniscus D. torn lateral collateral ligament E. torn medial collateral ligament
120) A 48 year old man arrives in the clinic complaining of chest pain and a persistent cough. You discover his cough is productive and has been producing blood streaked sputum. He has a 25 year history of smoking 4 packs a day. He does not drink alcohol and has no other significant clinical conditions. A likely cause for his symptoms would be:
A. cancer of the lung B. postnasal drip C. laryngitis D. Tracheobronchitis E. pulmonary edema from being a pink puffer
121) A man walks into the emergency room complaining of abdominal pain. He explains he has a steady pain radiating to his back. Upon physical exam, you find a low-grade fever, tachycardia, palpable upper abdominal mass, abdominal tenderness and hypotension. From the man's past history, you learn that he has had acute pancreatits. What are NOT possible risk factors associated with pancreatitis? A. Organ failure B. GI bleeding C. Pancreatic necrosis D. Diarrhea E. Blindness
122) A young female with major depression presents to the hospital believing that she is having a heart attack. Upon examination it is determined that she is having hot and cold flashes, trembling, paresthesia, and dizziness/vertigo. The most likely diagnosis is:
A. Generalized Anxiety Disorder B. Panic Disorder C. Obsessive-Compulsive Disorder D. Schizophrenia
123) A 40 year old woman presents with widespread musculoskeletal pain and tender joints. She describes her pain as being "all over," but especially in the neck, shoulders, hands, low back, and knees. She denies any swelling but has sensations of stiffness, especially in the mornings. Based on your knowledge of patterns of pain in and around joints, the most likely diagnosis is:
A. Rheumatoid arthritis B. Osteoarthritis C. Fibromyalgia syndrome D. Gouty arthritis E. Polymyalgia rheumatica
124) Which of the following represent the Cushing's Triad: 1)Systolic HTN (widening pulse pressure) 2)Increased ICP (Intracranial pressure) 3)Bradypnea 4)Bradycardia (bounding, slow pulse)
Answer Choices: a) 1, 2, 3 b) 1, 3, 4 c) 2, 3, 4 d) 1, 2, 4
125) While you are working in the ER a middle-aged man comes in complaining that "all of a sudden it was like something was ripping into the middle of my chest." He says the pain is severe and now spreading to his back. There is no feeling of chest compression and the pain feels the same whether he is sitting, standing, or laying down. His wife tells you he has a history of hypertension. Upon examination, you can barely feel his peripheral pulses. What diagnostic test should you order and what would you expect to see in the test results?
A. Lung scan; most likely result would be a ventilation/perfusion mismatch B. Chest x-ray; most likely result would be radiolucency within plural space C. Chest x-ray; most likely result would be a widened mediastinal silhouette D. Test arterial blood gas levels; most likely result would be hypoxemia E. ECG; most likely result would be PR segment depression
126) A mother brings in her 16 yr old son to your clinic. She claims that her son has become increasingly impulsive and manipulative with "hot and cold mood swings." She says that he lashes out with bouts of rage when he does not get his way, and it has gotten to the point that she fears for the safety of her family and for others. Which of the following personality disorders best describes the young man?
A) Cluster A - shizotypal B) Cluster B - narcissistic C) Cluster C - passive-aggressive D) Cluster B - borderline E) Cluster B - histrionic
127) Which of the following is not a clinical manifestation of adverse reaction to Rifampin?
a) hyperbilirubinemia b) hemolytic anemia c) cholestatic jaundice d) diffuse hepatocellular damage
128) A 55 year old man is referred to a neurologist by his primary care physician reagrding a speech disorder after suffering from a stroke. He presents with nonfluent, slow, and laborious speech difficulties. He is able to comprehend words, his repetition is impaired, he is able to recognize objects, and his reading comprehension is fair. What part of the brain suffered the lesion causing this type of aphasia?
A) Posterior superior temporal lobe B) Posterior inferior frontal lobe C) Posterior inferior temporal lobe D) Posterior inferior cerebellum E) Posterior superior frontal lobe
129) A 44 year old male presents with chest pain that he describes as a "burning" pain. He has found that this pain can be alleviated when he sleeps in a more upright position & that the pain is worsened by his morning caffeinated coffee & the large meals he eats throughout the day. He also explains that the pain is the worst shortly after he eats and sometimes will alter his sleeping habits if he eats too close to the time when he goes to bed. Which of the following is NOT considered to be one of the twelve components of the HPI?
a. pain in the chest region (Bodily Location) b. 44 y.o. male (Age & Sex of Patient) c. "burning pain" (Quality of Pain) d. sleeping with his head elevated relieves the pain (Relieving Factors) e. the pain is worsened by his morning coffee & large meals (Aggravating Factors)
130) While auscultating the lungs of your patient during a physical examination, you hear abnormally located bronchial breath sounds and decide to assess her transmited voice sounds. While listening, you ask her to say "ee," and notice that it sounds like "ay" through the stethoscope. The sound has a nasal quality. What is the name for this phenomenon?
A.) Broncophony B.) Whispered Pectoriloquy C.) Ayo Technology D.) Egophony E.) Ronchi
131) Occulsion of Anterior Cerebral Artery will show all of the signs and symptoms except:
a. Paralysis of leg and foot b. Cortical sensory loss over leg c. Urinary inconsistency d. Gait Apraxia e. Hyperactive reflex of the leg
132) Testing the pupillary reaction to light during a neurological exam involves the testing of which of the following nerve(s): a- cranial nerve II b- cranial nerve III c- cranial nerve IV
A) only a B) only a and b C) a,b, and c D) only a and c E) only b
133) What is the best diagnostic test to confirm Cholelithiasis?
a. MRI b. CT c. Ultrasound d. Serum Amylase e. Urinalysis
134) A 38 year old patient presents to your office with complaint of severe right ankle pain persisting for the past 3 months. The pain is most severe in the joint itself and no inflammation is evident. During the exam he also indicates that he has had mild hip and knee joint pain which began a short time after his years of playing college football. According to his symptoms this patient is most likely suffering from what disease?
A.) Chronic inflammatory polyarthritis B.) Chronic noninflammatory arthritis C.) Acute arthritis D.) Osteoarthritis E.) Bursitis
135) A 32 yo man comes into the emergency room complaining of anxiety, tremors, and blurred vision. Upon examination, you find he is tachycardic and is shaking. In his social history, you find that addiction runs in his family. Your primary diagnosis is:
a) alcohol intoxication b) narcotics withdrawal c) alcohol withdrawal d) schizophrenia e) major depression
136) A 14 yr old patient presents with wheezing, cough with sputum production, dyspnea, and fever for the last 3 days. He claims it gets worse during the night, and "is as if something is stuck in my throat". What is the most successful step it treating this patient?
A. Treat with B-adrenergic agonists B. Treat with Glucocorticoids C. Removal of Inciting agent (if possible) D. Desensitization or Immunotherapy
137) A 38 year old male comes in complaining of right upper quadrant pain and states that the pain has worsen. Patient relates to nausea and vomitting. Lab results show mild leukocytosis, an increase in serum bilirubin levels, as well as an increase in alkaline phophatase. What is the most likely etiology?
(a) acute pancreatitis (b) acute appendicitis (c) acute diverticulitis (d) acute cholecystitis
138) Patient comes in with complaints of irritability, nervousness, and difficulty concentrating. When questioned the patient notes of recent increase in appetite yet no weight gain. Which of the following would be the best exam for diagnosis of the indications?
A. Serum Blood Test B. MRI C. Nuclear Bone Scan D. Ultrasound E. Liver Function Exam
139) A 35 year old male presents with nausea and vomiting. Further examination reveals he has right upper quadrant pain, which sometimes radiates to the back. The pain often occurs 30-90 minutes after meals and lasts for several hours. Cullen's sign and Turner's signs are absent and there is no pleural effusion on the left. What should the next action be?
A) treat the patient for hypoglycemia (due to dysfunction of a specific organ) B) perform Murphy's sign to further confirm a diagnosis C) prescribe antibiotics and antacids D) give the patient a nasogastric tube E) perform a lung x-ray
140) A 48yo male changes insurance companies and presents to his new physician with no chief health complaint. During your visit, he mentions that he has recently lost his job, his home life is pleasant, and he feels like he's aging. He states that he "drinks a beer every once and a while, but since [he] lost his job, [he's] had more time on his hands." All of the following questions should be addressed except:
A) Do your family members/friends ever appear annoyed or criticize your drinking? B) Have you ever felt the need to cut back on your drinking? C) Do you wake in the morning in need of an eye-opener? D) Do you have a family history of Peyronie's disease? E) Do you ever feel guilty about drinking, or about your actions while drinking?
141) A 55 year old female patient presents to the emergency room with a broken arm after falling just outside the hospital. She is wearing several layers of winter clothing, despite it being mid-July and over 85 degrees outside. She has a tendency to repeat words and phrases that are said to her, seems surprisingly blasé for one with a broken arm, and often "loses her train of thought" midway through a sentence. Which psychiatric disorder is she most likely suffering from? A. Obsessive-Compulsive Disorder B. Major Depression C. Panic Disorder D. Posttraumatic Stress Disorder E. Schizophrenia
142) During a routine physical exam you are auscultating with the diaphragm of your stethoscope over the anterior lung fields and you hear a breath sound in the 1st interspace. As the patient completes one full breath, you notice that the inspiratory and expiratory sounds are about equal. You also note that the intensity and pitch of the expiratory sounds are intermediate compared to breath sounds from other areas of the lung field. Moving to the posterior lung field, this same breath sound can be heard between the scapulae. What type of breath sound is this consistent with? A. Tracheal B. Bronchovesicular C. Bronchial D. Rhonchi E. Vesicular
143) A 26 year old woman comes into your office complaining of wrist pain. She works as a secretary, and types on the computer all day. She experiences pain and numbness in the first three finger, especially at night, but has no problems with her palm. You suspect carpal tunnel syndrome, and decide to conduct a physical exam. Which one do you do?
1. Thumb abduction 2. Tinel's Sign 3. Phalen's Test
A. 1 only B. 1 & 3 only C. 1, 2, & 3 D. 2 & 3 only E. 3 only
144) The Emergency Department at your local hospital calls you in to perform a psychiatric consult on an elderly man who they believe is suffering from a personality disorder. After conducting your routine history and physical exam, you find out from the man's wife that he has been acting very avoidant, dependant, and obsessive compulsive. Upon further questioning about his obsessive compulsive behavior, you find that he isn't however displaying any signs of schizoid or paranoia-like behavior. Based on your thorough consult you diagnose the patient as suffering from what type of personality disorder?
A) Cluster A: Odd and Eccentric B) Cluster B: Dramatic, Emotional, Erratic C) Cluster C: Anxious or Fearful D) Major Depressive Disorder E) Anorexia Nervosa
145) A man presents at the emergency room. He has low cardiac output and has severe pain in his chest and back. When asked to describe the pain, the patient describes it as a "ripping" pain. Because of this quality of pain your initial diagnosis would be...
A. Myocardial Infarct B. Angina Pectoris C. Stroke D. Aortic Dissection E. Pulmonary Hypertension
146) 36 yo female bus driver PTC with a cc of "really bad headaches". She relates in a congested voice that she is having a hard time driving. She first noticed the headaches approximately 2 weeks ago. She relates that her pain is worse when she bends over or is supine. Her BP is 118/76; RR 18; PR 72; Temp 98.7. As you are taking the vitals you notice that she has thick purulent nasal discharge. From the information given what would be your be your most likely diagnosis.
A. Claudication B. Cancer C. Sinusitis D. Migraine E. Hemorrhage
147) A 47 year old female presents to your office complaining of shoulder pain. Patient relates to playing competitive tennis twice a week for the past 19 years. You proceed to perform a series of maneuvers to test specific muscles of the shoulder girdle and localize the pain. Patient relates pain as she abducts against resistance. What muscle is being tested with this maneuver?
a.) Subcapularis b.) Teres minor c.) Supraspinatus d.) Infraspinatus e.) Pectoralis major
148) A pt. presents to your office complaining of diffuse ankle pain. The History reveals the pt. has lateral ankle instability. Today he presents with mild edema, but no ecchymosis. Which of the following would be included in your differential diagnosis?
I - Osteochondral fragement II - Anterior talofibular ligament and Calcaneal Fibular Ligament injury III - Degenerative Joint Disease IV - Posterior Tibial Tendon Dysfunction V - Anterior Cruciate Ligament Injury
a. I, II b. I, III, IV, V c. I, II, IV, V d. I, III e. I, II, III, IV
149) A 45 year old male presents to the emergency room with sudden pain in the right upper quadrant of his abdomen, which seems to be getting progressively worse. You also note fever, nausea and two instances of vomiting. You highly consider a diagnosis of Acute Cholecystitis, though acute pancreatitis remains on your differential diagnosis. Which of the following would best help you rule IN Acute Cholecystitis?
A. Erythematous skin nodules B. Pain radiating to the right arm C. A positive Murphy's Sign D. You observe "caput medusa" E. Pitting edema in the legs
150) A 45 year old male presents to the clinic concerned about his recent weight loss and fatigue. He complains of "feeling tired all the time", despite sleeping 2-4 hours more than he used to. He has trouble concentrating at work, and quit his weekend softball league that he participated in for the last 15 years. He acknowledges that he has not been eating as much due to a decrease in appetite since his mother's death one month ago. He admits feeling "down" since his mother's death and denies thoughts of suicide. What is the best diagnosis for this patient?
A. Schizophrenia B. Panic Disorder C. Major Depression D. Obsessive-Compulsive Disorder E. Bipolar Disorder
151) A 45-year-old woman presents to her physician with an 8-month history of gradually increasing limb weakness. She first noticed difficulty climbing stairs, then problems rising from chairs, walking more than half a block, and, finally, lifting her arms above shoulder level. Aside from some difficulty swallowing, she has no ocular, bulbar, or sphincter problems and no sensory complaints. Family history is negative for neurological disease. Examination reveals significant proximal limb and neck muscle weakness with minimal atrophy, normal sensory findings, and intact deep tendon reflexes. The most likely diagnosis in this patient is
A. Polymyositis B. Cervical Myelopathy C. Myasthenia Graves D. Mononeuropathy multiplex E. Limb-Girdle Muscular Dystrophy
152) A 50 year old man came into the emergency room because his wife observed that his breathing patterns changed recently. He was shortly diagnosed with brain cancer (a megalloblastoma that compressed his pons). His new breathing pattern was a result of Central Neurogenic Hyperventilation. How would you describe the breathing pattern in this condition?
A. irregular with periods of apnea B. rhythmic with brief periods of apnea C. deep, very rapid respirations with no apnea D.gasping respirations with periods of apnea
153) The Monro-Kellie hypothesis states that an increaase in an intracranial component must be compensated by a decrease in one or more of the other intracranial components. Accordingly, all of the following might result from increased intracranial pressure except: a) Abnormal posturing where flaccidity leads to the decorticate position. b) An increasingly narrow pulse pressure range. c) A decrease in the level of consciousness from obtunded to stupor. d) Cushing's triad. e) none of the above.
154) A woman brings her 6 month old daughter into the emergency room. The baby appears to be wheezing and breathing very quickly and her chest is greatly expanding and retracting with each breath. Her skin appears white and upon touching her she feels cold. In addition, the child is not making eye contact, is crying, and is not easily consolable. Based upon physical diagnosis of symptoms alone, it is most likely that this child is suffering from:
A) Respiratory Distress B) Circulatory Shock C) Fever D) Respiratory Failure E) Hypothermia
155) A 17 year old female presents to the Emergency Room after injuring her left knee in a soccer game. You begin by assessing the patient's range of motion and discover the patient is unable to flex her knee past 20 degrees. You suspect an ACL injury. What would be the most appropriate test to perform next?
a) McMurray Test b) Anterior Drawer Sign c) Straight Leg Raise Test d) Lachman test e) Posterior Drawer sign
156) A 45 yr. old patient comes to the Scholl Foot and Ankle Clinic complaining of cramp-like pains in his right foot, particularly in the longitudinal arch. He has been a smoker since he was 18 years old. He says the pain cycles throughout the day and he is mostly on his feet; however, when he is sitting in his "lay-z-boy" chair at home, after work, his pain decreases. He's tried to quit smoking and realizes when he is not smoking during the day at work, his pain isn't quite as intense. You notice in his PE that his feet are sweating, cold, slightly cyanotic, and find two palpable lesions on the plantar aspect of his first and third distal phalanges. The probable diagnosis of this patient is:
A. Buerger's Disease B. Raynaud's Disease C. Superficial Thrombophlebitis D. Deep Venous Thrombosis E. None of the Above
157) A 24 year old female presents to a 24 hour clinic with complaints of nervousness, excess sweating, palpitations, weight loss with increased appetite, frequent bowel movements, and Plummer's nails. After a full physical examination you are considering a diagnosis of Graves' disease. Which of the following would help you rule in this diagnosis?
A. diastolic hypertension B. nonpalpable thyroid C. normal to raised TSH levels D. pale, doughy, cool skin E. enlarged thyroid with a bruit
158) A 42 year old female presents to your office with concerns of mild weight gain with poor appetite, lethargy, dyspnea and difficulty concentrating. You suspect hypothyroidism to be the underlying pathology. What feature would you most likely find upon physical examination that is consistent with this disorder?
A. Tachycardia B. Warm extremities C. Atrial fibrillation D. Prolongation of the relaxation phase of deep tendon reflexes E. Lid lag
159) 45 year old male presents to the emergency room with acute abdominal pain. Patient is hypotensive but hemodynamically stable. Abdomen is rigid, with increased bowel sounds. You notice that the sounds become high pitched when the patient complains of increased pain. Patient reports he had a cholecystectomy three months ago, but has not felt this type of pain before. What is the most likely cause of the patient's symptoms?
a) leaky abdominal aortic aneurysm b) gastroenteritis c) acute pleurisy d) mechanical bowel obstruction e) acute salpingitis
160) A 62 yr. old male smoker with type 2 diabetes and hypertension presents with a 4 month history of exertional chest pain. His physical exam was normal. During a stress test, he exercises for 8 minutes and experiences chest pain that radiates to the left side of his neck and down his left arm. An ECG during this test shows ST and T wave abnormalities. The most likely diagnosis for this patient is: a) Acute MI b) CHF c) Aortic anyeurism d) GERD e) Chronic stable angina
161) An 18 year old male presents to the Emergency Room in the middle of the night complaining of searing unilateral pain to the side of his face near his eye. He states that the pain was so bad, it woke him from sleep. He denies any nausea or vomiting, but states that he had a similar episode of pain at the same time last week. Upon further examination, it is noted that the patient is also experiencing some unilateral lacrimation. This patient is most likely affected by which of the following conditions:
A. Classic migraine B. Tension headache C. Trigeminal neuralgia D. Acute bacterial meningitis E. Cluster headache
162) You are examining a patient who presents to the emergency room looking slightly cyanotic, with bilateral wheezing on lung auscultation. The patient also demonstrates a prolonged expiration phase of breathing and use of accessory muscles to aid respiration. Treatment with a Beta-2 agonist is successful in alleviating the symptoms. Which of the following patients would be MOST likely to present this way?
A. 70 y/o male with previously unremarkable health history B. 56 y/o female ex-smoker C. 7 y/o male with known allergies D. 30 y/o female alcoholic E. 21 y/o female with previously unremarkable healthhistory
163) 24 year old female presents to your office complaining of digital ischemic changes of blanching, followed by cyanosis then rubor with cold exposure and rewarming. What disease (or phenomenon) could she possibly have?
a) Erythema Nodosum b) Raynaud's c) Schizophrenic episode d) Pitting Edema e) Colon cancer
164) A 38 year old patient presents to the physician's office with a sharp and localized pain in the right lower quadrant. What type is the pain and what body structure is affected? A)Visceral and intestinal pain of colonic origin B)Visceral and intestinal pain proximal to the ileocecal valve C)Somatic and external oblique inflammation D)Somatic and peritoneal inflammation E)Visceral and biliary pain
165) A 50-year old diabetic man presents to your office with chest pain. The episode occurred when he and his wife were on a hiking through the Grand Canyon during their vacation over the past week. He tells you that the pain only occurred after he had been hiking for a while, and it was relieved when he stopped and rested. When you ask your patient to describe the pain, he describes it as squeezing and radiating to his left arm. Your patient says that he is a smoker, but he has not been smoking as much in recent years. He reveals that his father was diagnosed with coronary artery disease at age 48. Upon physical exam, you find that his blood pressure is 143/90. You note an S4 sound while listening with the bell of your stethoscope.
Based on the presented information, your most likely diagnosis of this patient would be:
A. Aortic Dissection B. Non-ST-Elevation Myocardial Infarction C. Acute Myocardial Infarction D. Chronic Stable Angina E. Unstable Angina
166) You perform an MRI on a patient and find that they have a herniated disk. The patient also complains of numbness over the anterior portion of the knee and they have an absent patellar deep tendon reflex. At which spinal segment did the herniation occur? A- L5 B- L4 C- S1 D-S2
167) A 42 year old male presents to his primary care doctor complaining of fatigue, palpitations, and heat intolerance. Upon visual inspection the physician notes onycholysis, as well as pretibial myxedema. What would be the appropriate next step for the physian to take?
A. Order a CT B. Order a Thyroid Panel C. Perform a thyroid Exam D. Prescribe Propranalol E. Treat with radioiodine
168) A 37 year old female presents to the emergency department complaining of 8/10 chest pain that started ~ 1 hour ago while she was carrying boxes up the stairs. She describes the pain as constant and sharp with no radiation. She also has associated dyspnea. Patient denies syncope. The patient reveals a 20 pack/year smoking history, occasional alcohol use, no known medications allergies, she takes a multivitamin everyday as well as an oral contraceptive. She is single with no pertinent family history or past medical history. Patient just arrived from a cross country drive as she is moving to start a new job. Initial vital signs: BP 120/80; HR 130, regular; RR: 26, shallow. Which of the following procedures/tests would NOT be helpful in determining the cause of her pain?
A) Physical Exam including Heart/Lung and Musculoskeletal systems B) Electrocardiogram (EKG) C) Cardiac Enzymes D) Rinne and Weber Test E) Chest X-Ray
169) A 22 year old female patient presents to the Emergency Room with fatigue, loss of stamina, breathlessness, and tachycardia. Her hemoglobin (Hb) concentration is 10g/dL. This patient most likely has which of the following condition:
a. Moderate anemia b. Stroke c. Alcoholism d. Thyroid disease e. Syphilis
170) You have a patient that presents with symptoms of an aortic aneurysm which of the following would be true regarding your patient? I. The normal aortic pulsation is frequently visible in the epigastrium. An aneurysm is indicated by an increased pulsation or increased pulse pressure. II. Although an aneurysm is usually painless, pain may herald its most dreaded and frequent complication—rupture of the aorta. Apparent enlargement of the aorta indicates assessment by ultrasound. III. TREATMENT Control of hypertension is essential. Surgical resection of thoracic aortic aneurysms greater than 6 cm in diameter (abdominal aortic aneurysms greater than 5.5 cm), for persistent pain despite bp control, or for evidence of rapid expansion. In pts with the Marfan syndrome, thoracic aortic aneurysms greater than 5 cm usually warrant repair.
a. All of the above b. I and II c. II and III d. I and III e. None of the above
171) A 34 year old male patient presents to the Emergency Room with difficulty breathing. His blood pressure is 140/80, his temperature is 97.7 F, his pulse rate is 82 bmp, and his respiratory rate is 24 rpm. You notice that he has very deep, very rapid respirations with no apnea. This patient most likely has which of the following respiratory abnormalities.
A. Cluster Breathing B. Central Neurogenic Hyperventilation C. Ataxic Respirations D. Apneustic E. Cheyne-Stokes
172) All of the following are appropriate recommendations for women at high risk for heart disease, EXCEPT:
A. ACE-Inhibitors B. Hormone Therapy C. Beta-Blockers D. Daily low-dose aspirin E. LDL- lowering statin
173) A 35 year old woman complains of abdominal pains that have lasted for 4 days. She describes the pain as sharp, rated with an 8 on a 1-10 scale (with 10 being the worst pain ever). The pain has occurred every day, only in the afternoons and persists until the patient goes to sleep. She first noticed the pain after she went out for jog. The pain seems to be lessened with rest. This morning the patient began to vomit. She has experienced a similar sort of pain 2 years ago after being in an automobile accident, but denies a recent car accident. She is afraid that the pain may be due to appendicitis and is worried that she will be fired from her job if she has to miss a day of work. She already takes extra breaks at work and avoids physical activities.
What portion of an HPI is missing? A. setting/context B. hypothesis about the cause C. aggravating factors D. associated symptoms E. quality
174) A patient presents with severe rheumatoid arthritis. Which of the following would you NOT expect to be elevated or positive? A. Erythrocyte Sed. Rate B. C-Reactive Protein C. albumin D. thrombocytes E. rheumatoid factor
175) Which is not a primary cause of Hypothyroidism? a: I treatment b: Hashimoto's thyroiditis c: Iodine deficiency d:pituitary surgery e: atrophic thyroiditis.
176) A twenty-five year-old male medical student presents to the emergency room with a complaint of colicky periumbilical pain which has intensified over the last 6-8 hours and now has started to migrate to the right lower quadrant. The patient reports some initial nausea, and as the pain has increased she has had increasing emesis and anorexia. You suspect he has appendicitis. What is not a possible sign of appendicitis?
a. Rebound tenderness in right lower quadrant b. Positive Rovsing's sign c. Positive McMurray test d. Coughing or movement worsen pain e. Abdominal guarding
177) A second year podiatry student comes into the office complaining about being sleep deprived and stressed. The "game plan" for dealing with the stress would include: A. Thinking "out the Box" with regards to approaching stress B. Identifying options C. Sharing the stress D. Finding mental, emotional and physical outlets E. All of the above
178) Which of the following does not have the ability to cross the blood-brain barrier?
a) glucose b) antibiotics c) alcohol d) anesthetics e) oxygen
179) An excited, overly enthusiastic 19 year-old Caucasian female speaks in a rapid, pressured fashion when telling you stories about how much her boyfriend loves her. As she talks, she has trouble staying on one topic and she shifts rapidly from one topic to other superficially-related topics. Her family insists that her stories are false, and her "boyfriend" is actually just a classmate she calls repeatedly (and who never answers). Her family also reports that she stays up late at night doing miscellaneous projects, and that she claims not to need more than an hour of sleep per night. She has been highly energetic and her mood has been elevated since returning from an overseas family vacation last week; for months prior to that, her mood had been depressed. Her family denies any family history of similar mood changes. You're considering a diagnosis of Bipolar Disorder, Type I. All of the following in this patient's presentation are characteristic of this disorder, except:
A. Unusual talkativeness B. Delusional thinking C. Previous state of depressed mood D. Negative family history of mood disorder E. Decreased need for sleep
180) Clinical symptoms of substance abuse include all of the following except:
A. Withdrawl B. Persistent desire C. Greater time spent with the substance D. Depression E. Tolerance
181) A 47 year old man comes to the ER with the triad of symptoms of: jaundice, fever and right upper quadrant pain. The most likely diagnosis is:
a) cholangitis b) acute pancreatitis c) chronic hepatitis d) diverticulitis e) appendicitis
182) A man presents to his physician complaining of pain in his lower stomach. The man is 28 years old and has had bouts of anxiety. He is currently having a difficult time with his job and fears losing his wife. The man states that he has painful watery defecation. The man denies nausea and upon inspection does not present with herniations or saclike protrusion of mucosa. What is the most likely diagnosis for this mans affliction?
A. Diverticular Disease B. Intestinal Pseudoobstruction C. Irritable Bowel Syndrome D. Kwashiorkor E. Both A and D
183) A 30 year old female presents in your office complaining of hearing loss in her right ear, tinnitus, and headache. She also complains of numbness on the right side of her face. She has had these symptoms for approximately 5 weeks. Recently, within the last week or so, the headaches have gotten worse and been accompanied by nausea and vomiting. Furthermore, she also has felt unsteady on her feet has fallen down twice due to "feeling off balance". What is the BEST diagnosis?
A. Angioma B. Meningioma C. Right Acoustic Neuroma D. Left Acoustic Neuroma E. Astrocytoma 184) A 55 year old male patient has experienced a bought of bad luck. In the past month, he got a divorce from his wife of 25 years, lost his job, got into a car accident and is now in debt. On top of these circumstances, he has a negative outlook on life. Which one of the patient's issues can be classified as an internal stressor?
A.) He got divorced B.) He lost his job C.) He got into a car accident D.) He is in debt E.) He has a negative outlook on life
185) A 47 yr old female presents to your office complaining of pain to her right knee. She rates her pain as 7 out of 10. She relates that her knee swells after she takes her daily walk. She denies any similar pain in the left knee. She also relates that she hears a cracking sound when she flexes her right knee. Besides the fact that she has a history of peptic ulcers, her past medical history is unremarkable. Upon physical exam you notice firm nodules over the knuckles of her hands and swelling of the right knee. Upon passive ROM of the knee, you note an audible creaking with the right that is not present in the left. Radiographic examination reveals joint space narrowing. What is the best method of treatment for her? (pg 794-796)
A. Systemic glucocorticoids B. Topical capsaicin and NSAIDs C. Intraarticular hyaluronin D. Increase exercise until pain subsides
186) Osteoarthritis is the most common form of joint disease. Which of the following is the least likely joint to be involved:
A DIP joint of hand B PIP joint of hand C MP joint of hand D Knee joint E Hip joint
187) Patients who are in bed for a long period of time are prone to getting clots; the most common site to get clots is?
A- Lungs B- Heart C- Brain D- Lower extremity E- Upper extremity
188) A 56 year old male comes to the ER with a persistent fever, dyspnea, and cyanosis. He has a history of cardiac disease. The chart particularly indicate mitral stenosis. What might be a cause of his current symptoms:
A. cholangitis B. primary viral pnemonia C. irritable bowel syndrome D. right heart failure E. renal failure
189) A patient comes into your office complaining of pain in her hand. She has had this pain for over six weeks. You examine her hand and notice that all of her fingers are symmetrically inflammed especially at the PIP joints. What is the most likely diagnosis?
A) Osteonecrosis B) Rheumatoid arthritis C) Charcot arthritis D) Gout E) Reiter's syndrome
190) Which of the following is not included in the HPI? a) Quality of pain b) Primary Care Physician c) Location of pain d) Aggravating factors e) Onset 191) A patient comes to the hospital complaining of a unilateral throbbing headache that lasts usually between 2-6 hours. She noticies that these headaches occur after drinking wine or when she exercises; however, she finds relief after sleeping. The onset is fairly rapid and reaches a peak in 1-2 hours. She notes that she has had nausea and has been vomiting. What is your diagnosis? A) Tension Headache B) Migraine Headache C) Cluster Headache D) Headache from sinusitis E) Headache from a subarachnoid hemorrhage
192) Which of the following is the greastest risk factor for Suicide? A) Prior suicidal behavior B) Male (gender) C) Unwillingness to accept help D) Age E) Unemployed/retired
193) A 26 year old male presents to the clinic with a history of smoking 1 pack of cigarettes/day for the past 8 years. He has a family history of coronary artery disease, and says that he drinks about 3 beers every day. This patient is employed as an administrator for a local rental car company (sits behind his desk for most of the day) and gets very little exercise outside of work. His blood pressure is currently 138/86. What would you suggest for this patient first:
A. Start drug therapy immediately B. Recommend changes in his lifestyle like quitting smoking, greatly reducing his beer intake, and begin an exercise routine C. Perform risk factor screening for CHD at his next visit, and let him go home today without any medication or guidance on improving his lifestyle D. Do nothing, the patient hasn't complained of angina, and his blood pressure is considered a "high normal."
194) A patient presents to the E.R. with hesitant speech and word-finding difficulty which began an hour ago. The patients' eyes gaze preferentially to the right side. What OTC medication could have been taken if the patient believed they had suffered from a stroke? Aspirin (upto 325 mg/d)
195) A 35-year old Caucasian female presents to her primary care physician with feelings of despair, loss of appetite, fatigue and an increase in ecstasy use. She states that she began feeling this way three years ago and the symptoms reoccur every 7-9 months. Patient denied feeling periods of extreme happiness or joy. She is on interferon treatment for Hepatitis C.
This patient most likely has: a.) Generalized Anxiety Disorder b.) Major Depression c.) Obsessive-Compulsive Disorder d.) Bipolar Disorder e.) Schizophrenia
196) A patient presents to the clinic with pain in the upper right quadrant of their abdomen. The patient states that pain is worst after they have eaten and radiates to their back and scapula. The patient is diagnosed with cholelithiasis (gallstones). Which of the following methods would most likely have been used to make this diagnosis?
A. Plain film x-ray B. Sinography C. Computed Tomography D. Ultrasound
197) A female patient comes to your office because she is concerned about her health. Ms. Patient has a history of diabetes and and a mother who recently died of cardiovascular disease. Which of the following would you NOT recommend as a preventative medication for heart disease?
A) Aspirin B) Beta-blockers C) ACE Inhibitors D) Hormone therapy (combination of estrogen and progesterone) E) Angiotensin-receptor blockers
198) A 25 year old male patient tells his primary physician that aliens come to his house every night and visit with him. He also can hear their voices throughout the day. This patient most likely has:
A) Compulsions B) Depression C) General Anxiety Disorder D) Schizophrenia E) Phobias
199) A 35 year old male presents with complaints of muscle weakness, pain and stiffness. He admits that his brother has had similar symptoms, and both have suffered recurrent seizures throughout their lives. You order lab tests and a muscle biopsy, which shows the presence of ragged red fibers. You determine that the patient is most likely suffering from which type of myopathy?
A) Toxic Myopathy B) Mitochondrial Myopathy C) Metabolic Myopathy D) Inflammatory Myopathy E) Nemaline Myopathy
200) A 2 year old child presents in her father's arms with abnormal breathing and stridor, intercostal retractions, and nasal flaring. She does not appear normal. She is agitated and appears cyanotic. The child:
a. is in respiratory distress b. has an object lodged in her right lung c. is in respiratory failure d. is winded from running around on the playground e. has severe asthma
201) A patient arrives to his physician complaining of fatigue. Laboratory tests are unremarkable. The fatigue occurs upon exertion, and causes cramps. What is the most likely cause?
A) muscle disease B) neurogenic C) lipid D) cardiovascular E) a dystrophin complex disorder
202) A mother reports to your office with her 5 year old son, complaining that he has been less active especially with walking and lifting things lately. The mother reports to you a family history including Duchenne Dystrophy. Which of the following would establish this diagnosis in the 5 year old patient?
A) Atrophy of calf muscles B) Decreased CK and aldolase in laboratory findings C) Dystrophin in muscle tissue determined by Western Blot D) Worsening of symptoms after glucocorticoid therapy E) Presence of hammer toe deformity
203) A 49 year old male was admitted to the ICU yesterday for an atrial fibrillation. Today he is presenting with nausea, some vomiting, and periumbilical pain that is out of proportion to tenderness. You notice some distention of the abdomen and request an abdominal X-Ray, which clearly shows distention of the bowels. What is the likely diagnosis for this patients condition?
A. Pancreatitis B. Duodenal ulcer C. Consipation D. Acute mesenteric ischemia E. Really bad gas
204) What is the most common cause of cancer pain?
a. Cancer-directed therapy pain b. Migraine pain c. Directed nerve involvement pain d. Diabetic neuropathy pain e. Myofascial pain
205) Which of the following is NOT a Risk Factor for Suicide? a.)loss of physical health b.)age (45 and older) c.)family history of suicide d.)female e.)availability of means of suicide
206) Cheyne-Stokes breathing is characterized by A. Very deep, very rapid respirations, no apnea B. 2-3 second inspiratory and / or expiratory pause C. Irregular, random pattern; deep and shallow respirations with periods of apnea. D. Rhythmic; crescendo & decrescendo rate and depth of respiration; brief periods of apnea
207) This question comes from page 679 of Harrison's. What would most likely explain typical symptoms of fever, cough (either nonproductive or productive of purulent spututm), pleuritic chest pain, chills or rigor, and dyspnea? A. Influenzae B. Asthma C. Community Acquired Pneumonia D. Hospital Acquired Pneumonia E. Sinusitis
208) Which of the following psychiatric disorders is defined as a cyclical mood disorder in which episodes of major depression are interspersed with episodes of mania or hypomania? A. panic disorder B. obsessive-compulsive disorder C. phobic disorder D. bipolar disorder E. schizophrenia
209) A 75 year old male has been hospitalized for 3 days in the ICU after bypass surgery. He has a ventilator helping him breathe. The patient has had a fever for the past 2 days, he has pleuritic chest pain, and his white blood cells are elevated. A chest x-ray reveals a progressive infiltrate. What is the likely diagnosis for this patient?
a. infected incision from surgery b. hospital acquired pneumonia c. allergic reaction to pain medication d. sepsis e. not enough information
210) A 28 year old male reports to the emergency department with lower back pain and a fever. The patient relates that the pain does not get better when he rests. The patient states during his history that he has been an IV drug user for the past year. Laboratory results reveal an elevated ESR and an MRI reveals an abscess present. What is the probable etiology for the patient's back pain?
a. vertebral metastases b. lumbar disk disease c. vertebral osteomyelitis d. spinal stenosis e. trauma
211) At 3:00 in the morning an 18 year old male presents to the E.R. complaining of a searing pain on the left side of his head that seems to be coming from behind his eye. The patient related that he fell asleep fine but woke up 3 hours later with severe pain, unilateral lacrimation, and nasal and conjunctival congestion. The patient was concerned because two years ago he had the same type of headache every night for 6 weeks and then they just went away. What is the most likely diagnosis for this patient?
a. Tension Headache b. Severe Migraine c. Ruptured left middle cerebral artery aneurysm d. Cluster Headache e. Retroorbital tumor
212) A 55 year old women presents to your clinic complaining of pain in her hands and feet. She states that the pain is worst in the morning when she first wakes up, especially the first 30 minutes. She states that the pain is better when she is resting, but again, worsens when she begins to go about her day. She has been having trouble lately doing her normal daily activites, such as cleaning her house. She has also recently noticed changes in her fingers and toes (they are deviated laterlly). On exam, you notice crackling of the joints in her hand when you perform PROM and that she cannot fully close her hand into a fist on AROM. Her labs were all normal, but on X-ray you notice some osteophytes and joint space narrowing. What is most likely her diagnosis?
A. Rheumatoid Arthritis B. Gout C. Osteoarthritis D. Osteomyelitis E. Charcot Neuroarthropathy
213) A 32 year old male is brought to the emergency room after feeling intense pain in the substernal region. The pain is described as being of rapidly onset and of ripping quality radiating to the back. The pain does not go away by changing positions. The patient has never felt pain like this before. The attending also takes a history and notes some physical characteristics which include: height of 6 feet, 7 inches; weight 130 lbs.; long slender body and limbs; remarkably early onset glaucoma. The patient also describes himself as a "teetotaler". Which of the following problems do you attribute the current symptoms to? A) Parinaud's Syndrome B) Aortic Dissection C) Tumor in the Superior Cervical Ganglion D) Aortic Aneurysm at the Lumbar Plexus E) Mallory-Weiss Tear
214) A 45 year old Caucasian female comes to her primary physician's office, complaining about bilateral and similar muscle weakness in her proximal extremities, especially with her upper thighs. She complains about fatigue with walking. She also had difficulty raising her arms over her head. The muscle weakness has worsened over a course of 8-12 weeks. No skin rash was noted upon initial inspection. Laboratory findings shows that muscle enzymes were elevated 25 fold. Electromyographic findings revealed myopathy. Which of the following findings in the muscle biopsy is consistent with the patient's condition?
A. Ragged red fibers. B. Inflammatory Complexes with no vacuole. (I’m not a 100% sure) C. Sodium urate crystals. D. Beta-amyloid deposits. E. Perifascicular atrophy.
215) A middle-aged woman comes into your office complaining of pain in both hands and both feet for the last two months. Upon palpation and examination of the range of motion of the hand and feet joints, you do not hear anything out of the ordinary, but the PIP, MCP, and MTP joints are warm and tender to the touch. When you test for muscle strength, you find that the patient has a weak grip. What is the most likely diagnosis?
a) osteoarthritis b) rheumatoid arthritis c) synovitis d) tendinitis e) gout
216) After a long night of working in the ER, the attending physician asks you to examine one more patient before you go home to study for your USMLE Step I. Upon walking into the patient's room, introducing yourself, and extending your hand for a handshake, as taught to you by the ECR team, you notice hesitant speech with word-finding difficulty, preserved comprehension, anomia, and hemiparesis. What other finding would you expect to see?
a. Gaze preference with eyes deviated to side of lesion and hemisensory deficit due to a damaged MCA b. Gaze preference with eyes deviated to side of lesion and hemisensory deficit due to a damaged ACA c. Paralysis/paresis of vertical eye movement due to a damaged PCA d. Paralysis/paresis of vertical eye movement due to a damaged PICA
217) A middle-aged man comes in to your office complaining of pain in the epigastric region that radiates to the back. He relates to you that the pain is steady and nothing seems to help relieve it. He reports that it is often worse after a fatty meal or when he drinks alcohol. He does not report nausea or vomiting. What is the most likely diagnosis?
A. Cancer of the pancreas B. Peptic Ulcer C. Acute Pancreatitis D. Biliary Colic E. Chronic Pancreatitis
218) Your patient is a 70 year-old man with a long history of smoking who now has fever and a cough productive of greenish sputum. You suspect pneumonia and a chest x-ray confirms the diagnosis. If a Gram stain of the sputum reveals gram-negative rods and there is no growth on a blood agar but colonies do grow on chocolate agar supplemented with NAD and heme, which of the following bacteria is the MOST likely cause?
A Chlamydia pneumoniae B Legionella pneumphlia C Mycoplasma pneumoniae D Haemophilus influenzae E Streptococcus pneumoniae
219) A 57-year-old female presents to your office with a chief complaint of having "the worst headache ever." She explains that before bedtime last night she had felt nauseous and vomited. Shortly after, she fell asleep, but then awoke awhile later from the pain in her head. The pain is 10 out of 10 on the pain scale and is localized to an area directly above her left eye. She notices that it gets even worse with movement such as bending or lifting. She has never had severe headaches before and is concerned that it could be something serious. All of the following symptoms of this woman's headache suggest a serious underlying disorder EXCEPT:
a.) First severe headache b.) Onset after age 55 c.) Vomiting prior to the headache d.) Localized area of pain e.) Complaint of "worst headache ever"
220) 47-year-old woman comes into the office with high fever and cough with purulent sputum. She also complains of headache, nausea, vomiting, diarrhea, and fatigue. Physicial examination reveals tachypnea, dullness to percussion, increased tactile and vocal fremitus, egophony, whispering pectoriloquy, crackles, and pleural friction rub. The physician orders one imaging study to confirm his diagnosis. Which one of the following is most likely?
a) Chest x-ray b) Angiogram of the heart c) Knee MRI d) CT of the head e) MRI of the neck
221) A 52 year old female walks into your clinic and complains that she has had trouble hearing within the past couple of weeks. You perform a Weber test, and she says the sound is louder in her right ear. You then perform a Rinne test on her right side, where you discover that the the patient can hear sound longer when the tuning fork is on her mastoid process compared to when it is next to the external auditory meatus. What is the MOST likely cause of her hearing deficiency?
A) Lesion in her right inferior temporal gyrus B) Excessive cerumen in her right ear C) Hemorrhage in the right middle cerebral artery D) Lesion in the right cochlear nucleus E) Lesion in the right superior temporal gyrus
222) A 39 year old woman, comes in to your office with clinical features including persistent, excessive and or unrealistic worry associated with muscle tension, impaired concentration, feeling "on edge" and insomnia. She relates that she worries over minor matters. She denies having shortness of breath, palpitations, and tachycardia. The most probable diagnosis would be...
A.)Generalized Anxiety disorder B.)Panic Disorder C.)Depression D.)Obsessive-Compulsive Disorder E.)Somatoform Disorder
223) You are seeing a female patient in the clinic who complains of headaches that occur bilaterally. They also complain of nausea as an associated symptom. Which of the following statements is MOST true regarding this patient's condition?
a) Migraine Headache. b) The Maxillary Sinus is the most common location for Sinusitus to occur. c) Brain abscess is a potential life-threatening complication of this condition. d) Tension Headaches usually wake a person up in the middle of the night. e) Acute Sinusitus.
224) A middle-aged, slightly overweight man presents to the emergency room vomiting blood and complains of intense chest pain localized to substernal and epigastric regions. He tells you that his vomiting began recently and hasn't gone away. You perform a physical examination, where you note subcutaneous emphysema and hear crepitus adjacent to the sternum. At this point, you are considering the diagnosis of:
a) aortic dissection b) acute pericarditis c) peptic ulcer caused by H.pylori d) rupture of esophagus e) pulmonary embolism
225) Which of the following is NOT a common cause of increased levels of total T4 and T3?
a) pregnancy b) bile duct obstruction c) estrogens d) cirrhosis e) hepatitis
226) A lethargic 45 year old woman presents to her physician with dry hair and skin, difficulty concentrating, and mild weight gain with a poor appetite. She complains that she has been experiencing constipation, muscle cramping, and an inability to tolerate cold temperatures. After examination the physician notices bradycardia and a goiter. The most likely diagnosis will be:
A) Hyperthyroidism B) Hypothyroidism C) Thyroid Neoplasm D) Toxic Adenoma E) Sick Euthyroid Syndrome
227) A hulking 46 year old man comes into the doctor's office complaining of headaches and an overall apathetic feeling towards life. The man was also walking with a cane. After talking to the patient for a bit you find out that he used to be an offensive lineman for the Chicago Bears in the late 80's. Intrigued by this fact, you ask the ex-player what he thought about Mike Ditka. The former lineman tells you that Ditka was a great coach who inspired his players to play to levels they never thought they could and even play when they were hurt. The patient was no exception and tells you that he had played through injuries plenty of times throughout his career even though they had clearly taken a toll on his body and he couldnt understand why he was suddenly sad all the time and lost his zest for life. He used to be the life of the party but now he could barely get himself up out of bed in the morning. What is the most likely diagnosis?
a) Epilepsy b) Dementia c) Stroke d) Concussion induced depression e) Parkinson's
228) A patient in the ER presents with tachypnea and a long, labored, wheezing expiratory phase. The physical exam reveals breath sounds that are symmetrical bilaterally and tachycardia. Abnormal sounds during percussion are also found. A diagnosis of asthma is made. Based on this diagnosis, the abnormal percussion sounds were MOST likely: A) Hyperresonant B) Tympanic C) Dull D) Resonant E) None of the above
229) A 75 year old man comes in complaining of pain in the left knee. There is brief stiffness after rest or in the morning and physical examination reveals genu varum deformity. Routine lab work is normal, but joint fluid is straw-colored with good viscosity. A topical capsaicin cream was given to the patient. This patient was most likely diagnosed with which of the following:
a) Hydroxyapatite arthropathy b) Pseudogout c) Rheumatoid arthritis d) Osteoarthritis e) Gout
230) A patient has decreased movement of the chest on the left side, decreased fremitus, dullness to percussion, and decreased breath sounds on the left. The trachea is deviated to the right. The most likely diagnosis is:
A) Chronic bronchitis B) Pneumothorax C) Consolidated pneumonia D) Pleural effusion E) Atelectasis
231) Which of the following is a component of CAGE?
A) Anger B) Empathy C) Cut down D) Excited E) Grief
232) Which of the following is NOT the risk factor for suicide? a. Verbalized suicidal ideation b. Previous attempt or fantasized suicide c. Anxiety, depression, or exhaustion d. Family history of suicide e. eating disorder
233) A new patient comes to your clinic. The doctor asked where is your pain? Patient replied "I like main in the rain" what is the diagnosis of this patient?
A)Blocking B)Clanging C)perseveration D)ECholalia E)Flight of Ideas
234) A patient enters your office with a musculoskeletal complaint. The patient has been experiencing pain for about 8 weeks and he explains that he has been experiencing prolonged morning stiffness in the 2nd-5th digits of his right hand but the pain is not symmetric. You also notice slight soft tissue swelling in the affected joints. What is a possible diagnosis for his situation?
A) Rheumatoid arthritis B) Osteoarthritis C) Gout D) Psoriatic arthritis E) Acute arthritis
235) A patient comes in with complaint of diminished hearing. If this patient suffers form unilateral right ear sensorineural hearing loss then:
a.) Weber test shows sound lateralized to patient's right ear b.) Noisy environments make patient's hearing better c.) The patient's hearing loss is a result of acute otitis media d.) Rinne test shows sound is heard longer through air than bone e.) Weber test shows sound is heard equally in both patient's ears
236) A mother comes into your primary care practice with her six month old daughter. The child is crying, inconsolable, and cyanotic. You note that she is also grunting and has nasal flaring during inspiration. Based on these observations alone, you determine that she has an abnormal appearance and appears to be working harder and harder to breathe. You immediately determine that the child is most likely experiencing:
A. Shock B. Respiratory Failure C. Respiratory Distress D. Hunger E. A state that requires your continual observation only
237) A 56 year old man walks into your office with his wife. He has severe headaches, fever, and altered mental status. You suspect that the patient has meningitis. Which of the following tests do you do in order to confirm your diagnosis?
A. Kernig B. Babinski C. Brudzinski D. A and C E. A, B, and C
238) A 48 year old man presents with a steady epigastric pain that started suddenly. He describes the pain as radiating to the back and is persistent. He says that lying on his back makes the pain worse and it helps when he sits up leaning forward with his trunk flexed. He also reports nausea, vomitting, fever and abdominal distention. A history of alcohol abuse is noted. Which of the following conditions best describe the patient's symptoms? A)Peptic ulcer B) Cancer of the stomach C) Acute Pancreatitis D) Acute Cholecystitis E)Chronic Pancreatitis
239) A patient walks into a Physician office after suffering a stroke. The patient complains about lack of sensation and lack of movement on one half of her body. She is also experiences difficulty speaking and and understanding conversations. It could be seen during examination that she was experiencing impaired vision on the right part of her visual field. Based on these symptoms locate the region of the stroke.
A.) Posterior Cerebral Artery B.) Basilar Artery C.) Anterior Cerebral Artery D.) Middle Cerebral Artery E.) Vertebral Artery
240) A 57 year old man presents to the ER complaining of headache, visual problems, during your conversation with the patient, you note that he seems to be haveing trouble speaking, specificley finding the right words. you conclude the he has had a stroke. what artery of the brain would most likley be occluded?
A. middle cerebral B. posterior cerebral C. left internal carotid D. Basilar E. vertebral
241) A 43-year old man goes to the emergency department complaining of severe headache, stiff neck and fever that has lasted for three days. On examination, the patient displays nuchal rigidity and Brudzinski's sign. Which of the following is the most likely diagnosis? A. Hemorrhagic stroke B. Meningitis C. Embolic stroke D. Guillain-Barrй syndrome E. Sinusitis
242) A 35-year old male complains of burning pain in the epigastric region. He mentions that this often occurs a couple hours after meals and at night. He also says that the pain is relieved by eating something. What is the likely diagnosis.
A. gastric ulcer B. cancer of the stomach C. duodenal ulcer D. acute pancreatitis
243) A 37 year old female comes in complaining of pain, tingling, and numbness in her fingers and toes. She says that they only hurt when it is cold and that the pain comes and goes, but only lasts for a short period each time. Also, that the only way to make it better is to warm up. Luckily, it is a very cold day outside and she has just come in. You quickly examine her fingers and toes to find that they are extremely pale at the tips. What is the best diagnosis, given these symptoms?
A. Acute arterial occlusion B. Chronic venous insufficiency C. Arteriosclerosis obliterans D. Raynaud's disease E. Buerger's disease
244) A patient comes into your clinic looking very sleepy. He is not responding to your stimuli very quickly, but does respond. He eventually responds to your motor and mental commands in a sluggish manner. Your patient's consciousness level category can be defined as:
a) alert b) obtunded c) stuporous d) lethargic e) comatose
245) A patient comes to your office complaining of impaired pain and thermal sense over half of the body. Upon examination you notice vertigo, nystagmus, ataxia, and Horner's syndrome. He falls to his left side. Which cerebral artery is this patient likely to have a lesion in?
A. left vertebral artery B. right vertebral artery C. left anterior cerebral artery D. right anterior cerebral artery E. left posterior cerebral artery
246) A 54 year old patient presents to your office complaining of a painful first metatarsophalangeal joint. Looking at the chart you see the patient has a history of acute gouty arthritis. You inquire about the onset of the current symptoms. What is least likely to be one of your patients responses?
a. After a surgery to the metatarsophalangeal joint. b. When the patient has been consuming more alcohol than usual. c. After jamming his toe against the couch. d. While spending a long day constantly on his feet. e. Suddenly during the night.
247) A couple brings their 5 year old son into the emergency room. They are worried because he has been falling down repeatedly, does not play with the other children like he used to, and complains frequently of muscle cramps in his legs. From close examination you find increased muscle mass in his calves (despite the muscle weakness in his legs). What do you suspect is wrong with the child?
a. diabetic neuropathy b. multiple sclerosis c. polio d. Duchenne form of muscular dystrophy e. malnutrition
248) Which of the following is not true about Osteoarthritis?
A) Osteoarthritis is another term for degenerative joint disease B) It is characterized by progressive deterioration and loss of articular cartilage C) it is a very common form of joint disease D) joints with osteoarthritis will have pain free range of motion without crepitus or deformity E) Osteoarthritis can affect almost any joint, but most often occurs in weight-bearing and frequently used joints
249) A 16 year old male on the high school football team presents to your office with pain in his right knee. He says that his knee started to hurt after being tackled in a football game 3 days ago. You want to test his ACL for possible tearing. What test(s) would indicate a possible tear of his ACL?
1. Lachman Test 2. Anterior Drawer Sign 3. McMurray Test
a) 1 only b) 2 only c) 1, 2, and 3 d) 1 and 2 only e) 2 and 3 only
250) A 29 year old male comes to the clinic depressed. He states that for some time now, he has had trouble maintaining healthy relationships with family and friends. He states that in general, his daily life is unsatisfactory. During the history, the patient reveals he is impulsive, has a fear of abandonment, and has occasional rage episodes that he simply can't control. This patient could be best diagnosed with what type of personality disorder?
A. Histrionic Cluster B B. Compulsive Cluster C C. Borderline Cluster B D. Antisocial Cluster B E. Avoidant Cluster C
251) A 16 year old male on the high school football team presents to your office with pain in his right knee. He says that his knee started to hurt after being tackled in a football game 3 days ago. You want to test his ACL for possible tearing. What test(s) would indicate a possible tear of his ACL?
1. Lachman Test 2. Anterior Drawer Sign 3. McMurray Test
a) 1 only b) 2 only c) 1, 2, and 3 d) 1 and 2 only e) 2 and 3 only
252) A sixty-five year old woman comes to the hospital emergency room because of fever and abdominal pain. She is concerned she may have contracted an illness while on a trip to Mexico seven weeks ago. On physical examination the resident notes left lower quadrant abdominal tenderness, temperature taken orally of 99.8 degrees Fahrenheit, BP 108/52 mmHg. Lab results show a white blood count of 11.5 (normal 4.5-11), sodium 160 (normal 136-145 mEq/liter, chloride 108 (normal range = 98-106 mEq/liter), BUN 23 (normal 7-21). Urinalysis is not significant except for a specific gravity of 1.039 (normal 1.002-1.035). The patient states she has had alternating constipation and diarrhea and recurrent lower left quadrant pain for almost two years, which is relieved by defecation. She admits her eating habits are not very good since her husband died two years ago. Since that time, she says she "has no one to cook for" and eats fast food every day for lunch and dinner. She also mentions she does not like to eat fruits and vegetables nor drink a lot of fluids. She drinks two gin and tonics every evening to help deal with loneliness.
Based on the information above, what is the most likely diagnosis:
A.) Glomerulonephritis B.) Appendicitis C.) Pancreatitis D.) Diverticulitis E.) Salmonellosis
253) A 65 year old obese man with past medical history of Peripheral Vascular disease, HTN., diabetes presented to the emergency room with severe chest pain radiating to the back and abdomenand with 10/10 severity. He complains of sudden onset of pain which started 6 hours ago. On examination, pt. had a heart rate of 120, BP of 190/110 mmHg, abdominal tenderness, no rebound tenderness. Most likely diagnosis is? A: MI B: Aortic dissecting aneurysm C:Eosophageal rupture D:Tension Pnuemothorax E: Cardiac Tamponade
254) All the following are correct for the Visual Analogue Scale (VAS) except
A) Its a simple robust pain measurement tool B) It can be measured to severity and or improvement C) It is usually designed as a 10 cm line with descriptors at both ends D) It can not be used for children above the age of 5
255) What is the proper order for examinating the abdomen?
a.)inspect, auscultate, percuss, palpate b.)auscultate, inspect, percuss, palpate c.)percuss, palpate, auscultate, inspect d.)inspect, percuss, auscultate, palpate e.)none of the above
256) Which of the following is not a symptom associated with alcohol withdrawal system?
A) sleep cycle disturbance B) agitation C) disorientation D) tachycardia E) confusion
257) A 27 y.o. male presents to primary care physician with 24 hour history of malaise, fever, headache and chills. Pt denies any respiratory or GI symptoms, neck rigidity. Pt had a flu shot about 5 months prior and states that his symptoms are not as intense today as they were yesterday. What is the most likely diagnosis? A) Sinus headache B) Viral meningitis C) pneumocystis carinii infection D) influenza E) Rhinovirus infection
258) A 55 year old male presents at your office for his yearly check-up. The nurse records his blood pressure as 165/95 mm Hg. According to current standards this blood pressure is determined to be:
A. Normal B. Isolated systolic hypertension C. Prehypertension D. Stage 1 Hypertension E. Stage 2 Hypertension
259) A 40 year old woman comes to you with deep, steady epigastric pain which radiates to her back. She notes that the pain gets worse after eating a large meal. What other symptom will help you confirm your diagnosis? (bates pg 394)
A) fever B) vomiting C) constipation D) diarrhea E) obesity
260) A 50 year old presents with pain and tenderness in his calf. He complains his shoes fit tightly now as well. What is your diagnosis (bates pg 492)?
A) superficial thrombophlebitis B) deep vein thrombosis C) acute cellulitis D) congestive heart failure E) bacterial infection
261) Patient presents with low TSH, high free T4, negative for TPO antibodies, tachycardia, heat intolerance, and no ophthalmopathy. What is the best treatment option?
a) Partial thyrodectomy b) Combination of carbimazole and methimazole c) Levothyroxine d) Radioiodine e) Measure T3 levels
262) A 45 yo male smoker complains of having chest pain. He has a BP of 155/90, a family history of diabetes. Upon auscultation you hear bruits, rales and a loud S3. He has mentioned to you that he has been under a lot of emotional stress lately. An ECG shows abnormal T-waves. What is the most likely diagnosis?
A. Myocardial Infarction B. Chronic Stable Angina C. Aortic distention D. GERD E. Congestive Heart Failure
263) Krystal, a 35 year old woman presents to the emergency room complaining of right knee pain. She states that while skiing a particularly difficult run that afternoon, she felt her right knee "buckle" and heard a "popping sound." The knee is tender to palpation and swollen. The patient has a positive anterior drawer sign. You suspect a tear in which ligament?
A: medial collateral ligament B: lateral collateral ligament C: posterior cruciate ligament D: anterior cruciate ligament
263) As a fourth year medical student, you encounter a patient brought in for an evaluation. Which of the following would be INCORRECT in making a careful assessment of alcohol abuse and dependence? a.) Careful history and mental status exam b.) Use of information from family members c.) Blood alcohol levels d.) Use of SCUM- screening test e.) None of the above
264) A 50 year old female patient at your clinic presents as progressive and symmetric muscle weakness. She complains about pharyngeal weakness. Physical exam finds her extraocular muscles are unaffected. Lab results show that her muscle enzyme activity increases 50 fold. Muscle biopsy findings include inflammation with the CD8/MHC-I complex but no vacuoles. What is the most likely diagnosis? A, Polymyositis B, Dermatomyositis C, Includsion Body MYositis D, Duchenne Dystrophy E, Myotonic Dystrophy
265) Which is the best test used to CONFIRM the diagnosis of pancreatitis? a. serum amylase b. abdominal radiographs c. CT d. ultrasound e. serum lipase
266) A 53-year-old man presents with weakness of his left arm and left leg. He seems unsettled and generally confused. A CT scan shows an bleeding in his right brain. Diagnosed with an acute stroke, what is his likely blood pressure?
1) 80/60 2) 120/80 3) 160/100 4) 100/90 5) 90/70
267) When taking vital signs on a 54 year old male, you notice frequent premature contractions. What patient would experience such contractions? A. The anxious patient B. The obese patient C. The hypertensive patient D. The arrhythmatic patient
268) A 40 year old woman presents to the clinic complaining of recurrent abdominal pain on her right upper abdomen which radiates to her right shoulder blade and right upper back. Attacks present as severe steady, dull aches and wax and wane over a course of 1-3 hours and tend to occur within 30 minutes after eating a meal, especially with fatty foods. What is most likely the cause of this pain? A. Appendicitis B. Cholelithiasis C. Irritable bowel syndrome D. Gastroesophageal reflux E. Peptic Ulcer Disease (PUD)
269) A 45 year old male comes into your office complaining of chest pain. Using your stethoscope, you auscultate for abnormal heart sounds and murmurs, but you do not hear any abnormalities. However, your history strongly suggests mitral stenosis. What position could you place your patient in order to accentuate a possible mitral stenosis?
A) Sitting up, leaning forward, exhaling completely B) During a valsalva maneuver C) Left lateral decubitus position (patient is rolled onto his left side) D) Standing up E) Sqautting
270) A 40yr old male comes into the clinic for a concern he has with his condition. He confides in you saying he cannot get much sleep lately due to the fact that he is constantly checking if the front door is locked. He states that this probably occurs about 10 times a night. His history states that he has no other pre-existing conditions...Which of the below follows with the symptoms?
a. Schizophrenia b. Phobic disorder c. Major depression d. Obsessive compulsive disorder e. Anemia
271) Gertrude, your new patient is seeing you for foot and ankle pain relating to a fall from a fight. While describing you the situation, she explains that the tooth fairy was trying to steal her teeth to meet her monthly quota. As you probe for more information, she tells you that she is the fairy god mother and works alongside the santa claus, the tooth fairy, and that she regularly uses her magic wand to grant wishes to good people, and then offers you a wish. You, after referring her to a podiatrist, suspect the following personality disorder:
a) Axis II: Cluster A Personality Disorder b) Axis II: Cluster B Personality Disorder c) Obsessive-Compulsive Disorder d) Axis II: Cluster C Personality Disorder e) Post Traumatic Stress Disorder
272) A 35 y/o female present to clinic complaining of difficulty in sleeping, feeling tired all the time, lost 10 lbs in about two months. She also had recurrent thoughts of suicide. What is the most likely diagnose? a. Schizophrenia b. Phobic disorder c. Major depression d. Obsessive compulsive disorder e. Anemia
273) A 48 y/o male presents to the emergency room complaining of sharp, crushing pain in his chest. He says the pain is worse when he lays down and is relieved when he leans forward. He is lethargic and states that he is recovering from a cold. Upon examination, you notice a course pericardial friction rub. What is the etiology of the patient's chest pain? A. Cardiac Tamponade B. Pleurisy C. Acute Pericarditis D. Ruptured Esophagus E. Acute Myocardial Infarction
274) A 22 y/o male presents to the emergency room with pain that started near the umbilicus and now radiates to the right lower quadrant. The pain is aggravated by coughing and there are signs of muscular rigidity. What is the most likely etiology?
a) Colon cancer b) Appendicitis c) Salpingitis d) Cholecystitis e) Peyronie's Disease
275) A 65 year old male presents to the ER after an abrupt onset of focal neurologic loss. After history and examination of the patient, you find that he has paralysis and sensory loss over his legs and feet, accompanied by urinary incontinence and an apraxic gait. In your differential diagnosis, you suspect an ischemic stroke. What is the most likely vascular location of the attack?
A. Vertebral Artery B. Basilar Artery C. Anterior Cerebral Artery D. Anterior Inferior Cerebellar Artery E. Middle Cerebral Artery
276) A 21-year old female was brought into your emergency room with an altered level of consciousness. You speak to her in a loud voice. She is drowsy, but opens her eyes, looks at you, responds to questions, and then falls back asleep. How would you describe this level of consciousness?
A: alert B: lethargic C: obtunded D: stupor E: comatose
277) A 38 y/o male presented to the emergency department complaining of intense pain in his upper abdomen, and vomiting. The patient was slumped-over and did not look well. History revealed two previous episodes of acute pancreatitis, substance abuse treatment at age 22. Pt. related he has been compliant in taking pancreatic enzymes with meals, and that he drank only one beer the previous night. Labs revealed increased serum amylase and serum lipase and increased triglycerides. Ultrasound ruled out gallstones and CT scan revealed inflammation and destruction of pancreatic tissue. The diagnosis of acute/chronic pancreatitis was made. The MOST likely etiology is
A) Cholelithiasis B) Hereditary C) Unknown D) Alcoholism E) Metabolic
278) Pedro M., a 36 year old gentleman presents to clinic with weeklong feelings of malaise, as well as general indifference towards things he used to enjoy immensely. Physically, he hurt his leg not too long ago, but he insists, that's not why he has come in to see you. His concern is with how often he finds himself lethargic, and how he has a hard time "getting out of bed in the morning." When taking his history, you find that this past year, he drinks alcoholic beverages daily ("I usually have a beer with lunch and dinner...and even sometimes with his mango breakfast...just to feel better...") and as of recently he's had to drink at least 2 beers just to feel it. You also seemed to notice how visibly anxious he was when talking about how he has to take extended time off of work after his hurting leg, and how no one at work seems to miss him because of a new, younger employee that they just hired.
He, however, has never fantasized suicide, nor does he have feelings of hopelessness.
With this patient, : A) A definitive diagnosis of alcoholism can be made solely on the basis of the careful history and mental status exam. B) A definitive diagnosis of depression can be made solely on the basis of the careful history and mental status exam. C) A definitive diagnosis of alcoholism and depression can be made solely on the basis of the careful history and mental status exam. D) A definitive diagnosis cannot be made. E) A definitive diagnosis can only be made after taking his physical
279) A patient presents to the ER and while you are assessing his vitals, you notice that he has abnormal breathing patterns. You notice brief periods of apnea. Also, the breathing is rhythmic with crescendo and decrescendo rate and depth of respiration. What is this pattern of breathing called?
a) Cheyne-Stokes b) Apneustic c) Cluster breathing d) Ataxic breathing e) Central neurogenic hyperventilation
280) During your third year, you test the corneal reflex on a patient by lightly touching the cornea of the right eye with a wisp of cotton as the patient looks forward. You notice that there is absence of blinking in both eyes in response to the cotton wisp touching the right cornea. Which of the following is the MOST likely cause?
a) Occlusion of the Left Anterior Cerebral Artery b) Lesion of the Right CN II c) Lesion of the Right CN V d) Lesion of the Right CN VII e) Occlusion of the Right Anterior Cerebral Artery
281) An extremely thin 15 year old girl presents to your emergency with intense abdominal pain in the periumbilical area. The pain has lasted for approximately 4 hours. She complains of nausea and vomiting. You suspect that this patient has acute appendicitis. Which symptom does NOT suggest this diagnosis?
A. Positive Murphy's Sign B. Positive obturator sign C. Postive psoas sign D. Positive Rovsing's sign E. Referred rebound tenderness
282) You are performing an routine eye examination on a patient. As you exam their right eye with the ophthalmoscope, you notice small, rounded, irregular red spots near the retina. What is this an indication of and what could cause this finding?
a. superficial retinal hemorrhage; severe hypertension b. preretinal hemorrhage; increased intracranial pressure c. microaneurysm; diabetic retinopathy d. deep retinal hemorrhage; diabetes mellitus e. neovascularization; late proliferative stage of diabetic retinopathy
283) Which of the following is not a risk factor for suicide? A. Age (45 or older) B. Alcohol dependence C. Female D. Prior suicide behavior E. Unwilling to accept help
284) A 45 year old male comes to you with midepigastric pain that is radiating to his back. He says that the pain is made worse when laying on his back. He has had nausea and vomiting as well. He admits to drinking 6 days/week. Upon physical examination, a low-grade fever, hypotension, abdominal tenderness and a palpable mass in his upper abdomen were found. Which of the following is the most likely diagnosis?
A) Acute pancreatitis B) Acute intestinal obstruction C) Acute mesenteric ischemia D) Aortic dissection E) Pneumonia
285) A female patient comes to your office complaining of headaches with premonitory visual sensory symptoms, unilateral throbbing headache, nausea, and vomiting. She also complains of photophobia, and these attacks lasts about 2 hours. She also states that sleep usually relieves these headaches. Which type of headache would she be classified with?
A. Common Migraine B. Classic Migraine C. Tension-type headache D. Cluster headache E. Headache associated with head trauma
286) A 70 year old female develops sudden dyspnea after surgery. She complains of chest pain, cough with possible bloody sputum, and slight fever. No other symptoms present. What is the most likely cause of her symptoms?
a) mitral stenosis b) pulmonary embolism c) spontaneous pneumothorax d) myocardial infarction e) pericarditis
287) A patient presents to your office with digital ischemic changes of blanching followed by cyanosis in her hands and feet. There is also redness and inflammation with cold exposure and rewarming. She states that her fingers and toes become cold and go numb when she is under extreme stress. Which of the following diseases do you suspect your patient has?
a. venous stasis ulcer b. arterial peripheral vascular disease c. patient is normal d. Raynaud's disease e. edema
288) A 70 year old female patient comes to your office because of trouble with sleeping. You take note that the patient is well dressed, groomed, and makes appropriate eye contact. In addition to these normal attributes the patient seems to have normal motor coordination and gait. During the exam you wish to test the patients ability to concentrate. Which of the following tasks would NOT be used to test the ability of the patient to concentrate?
A) Naming presidents B) Digit Span C) Serial seven's D) Spell words backwards E) Repeat 3 words given to the patient immediately
289) A 54 year old woman comes into the ER complaining of shoulder stiffness and malaise. The patient admits to weight loss and describes the pain as being the worst in the morning. Upon inspection, there does not appear to be any swelling or redness of the shoulder joint. There also doesn't appear to be any loss in the patient's range of motion or strength compared to the contralateral shoulder. What is the most likely cause of these symptoms?
A. Rheumatoid Arthritis B. Rotator Cuff Tendinitis C. Adhesive Capsulitis D. Polymyalgia Rheumatica E. Calcific Tendinitis
290) 72 yr. old female from a local nursing home presents to your office in mid January complaining of cough (productive of purulent sputum), fever, pleuritic chest pain, chills, and headache. Upon physical exam you note tachypnea, dullness upon percussion, increased tactile and vocal fremitus, and crackles. Her vitals are: BP is 120/80, resp. rate is 34 breaths per minute, temp 99.8 degrees, pulse is 85 b.p.m.
What is this patients most likely diagnosis?
A. Chronic Bronchitis B. Chronic obstructive pulmonary disease (COPD) C. Influenza D. Community –Aquired Pneumonia (CAP) E. Hospital –Aquired Pneumonia (HAP)
291) A 55 y.o. Hawaiian native female presents with lethargy, cold intolerance, and mild weight gain with poor appetite. Lab results show decreased serum T4 with an elevated TSH. What is the most likely diagnosis?
A-parasitic infection B-primary hypothyroidism C-menopause D-secondary hypothyroidism E-autoimmune-mediated hypothyroidism
292) A patient of yours has been staying in the hospital following a metatarsal resection. On the fourth day of his stay he starts experiencing profound autonomic hyperactivity, extreme confusion, agitation, visual and tactile hallucinations, and vivid delusions. There is nothing in his records that would suggest such symptoms, however, you are an exceptional physician, and order a high dose of benzodiazepine. This patient is most likely suffering from:
a. schizophrenia b. delirium tremens c. hypothyroidism d. right bundle branch block e. acute MI
293) A patient walks into the ER complaining of abdominal pain, and tenderness. He is awake and has no signs of internal or external bleeding (hemodynamically stable). Upon inspection & palpation of his abdomen you notice that it is very rigid. What is the most probable cause? And what should be done next?
A. Perforation; chest and abdominal radiograph. B. Abdominal aortic aneurysm; immediately sent for surgical exploration. C. Obstruction; chest and abdominal radiograph. D. Odynophagia; videofluoroscopy of swallowing. E. Both answers A & C.
294) A patient comes into the E.R complaining of pain in the lower right quadrant of their abdomen. You suspect it could be appendicitis. Which of the following would help confirm your diagnosis?
a) Positive Psoas sign b) Negative Obturator sign c) Positive Rovsing's sign d) Positive Murphy's sign e) None of the above
295) A 25 y.o. male presents to your clinic with mood changes, anxiety, depression and general feeling of unhappiness. You ask him a few questions and determine that he is under a lot of stress and his daily diet and routine have been severely affected. Which of the following remedies would you suggest? A) Increase junk food intake B) Eat as fast as possible C) Increase caffiene intake D) Spend more time working E) Get regular exercise and sleep
296) 57 year old male complains of numbness and fatigue in his legs and feet. He is a heavy smoker. He has hypertension and hyperlipidimia. His father died of a stroke. He denies previous diagnosis or family history of diabetes mellitus. He denies exercise-induced leg pain, and claims no cramping or aching in the lower extremity. He denies polydipsia, polyphagia, polyuria and muscle weakness. On examination, his feet were cold, he is found to have decreased proprioceptive and vibratory sensation, and a loss of hair growth on the anterior tibilal surfaces. His ABI was measured at 0.85. What is the most like cause of this patient's chief complaint?
1) Spinal Stenosis 2) Peripheral Neuropathy secondary to Peripheral Arterial Disease 3) Restless Leg Syndrome 4) Type II Diabetes Mellitus 5) Superficial thrombophlebitis
297) A 40-year-old man is in the emergency room with complaints of severe aching in his left eye. Although he has been vomiting, there has been no blood in his vomit and it has not been associated with eating. However, he describes a feeling of nausea. History reveals that he has no previous history of headache, and the chief complaint has arisen within the last few hours. Physical exam shows visual abnormalities are absent besides some blurriness, and no exopthalmos; normal accomodation, pupillary reflex and corneal reflex. Lab findings indicate normal WBC count. Which of the following is most likely to be causing the chief complaint?
A) Acute bacterial meningitis B) Hyperthyroidism C) Migraine headache D) Acute glaucoma E) Detached retina
298) A 55 yr. old male admits to the emergency room with chest pain radiating to the neck and back which he describes as sharp and 8 out of 10. He admits to a 20 pack year smoking history. He describes palpatations and shortness of breath with the chest pain. No edema is present. His BP is 135/98 Which of the following is most likely the cause of the chest pain?
A. Angina pectoris B. Aortic dissection C. Myocardial infarction D. Congestive heart failure E. Gastroesophageal reflux disease
299) Patient presents with fever, and complains that they haven't been feeling well. BP is 85/55, RR is 23. Has a history of type II diabetes and a 20 pack history. Upon physical examination an ulcer is noted under the head of the first metatarsal. Area around the ulcer is swollen, with red streaks extending up the medial aspect of the leg. Popliteal lymph nodes are found to be normal, however femoral lymph nodes are enlarged. What would be the appropriate course of action.
A. Chest XRay B. MRI C. Saline and IV antibiotics D. Insulin E. Debridement of wound
300) 30 year old male presents with complaints of irritability, heat intolerance and heart palpitations. Pt also indicates that he has been losing weight recently. Which of the following diagnosis coincides with his symptoms?
A. Thyrotoxicosis B. Hypothyroidism C. Acute food poisoning D. Pulmonary embolism E. Syphilis
301) You are doing a patient history on a 25 year old female. During the interview you continue to have trouble getting complete answers from the patient and must repeat the questions. As you ask questions, she is able to respond but often stops abruptly in midsentence and not able to finish the response. She simply states that she loses her train of thought. What is the psychological condition?
A. Incoherence B. Blocking C. Anxiety D. Flight of ideas E. Echolalia
302) A 51-year-old female flight attendant presents to your office complaining of pain and swelling in her left leg. The patient states it developed suddenly two days ago after she returned to Chicago after a flight from New Zealand. While abroad she was walking around without pain or shortness of breath. She has a surgical history of a bunion correction five years ago, otherwise unremarkable. She denies alcohol, tobacco or drug use. Upon examination her left lower leg is 4 cm larger than her right with pitting edema, but no discoloration or change in temperature. Positive Homan's sign. Which best describes her condition?
(A) Erythema Nodosum (B) Deep vein thrombosis (C) Raynaud's Phenomoenon (D) Acute Lymphangitis (E) Intermittent Claudication
303) A 45 year old male complains of a recurrent headache. The headache is always in the same location and is made worse when the patient suddenly moves his head, coughs, or sneezes. The headaches are dull, often begin when he is waking up, and are often brief in length. The patient is not on any prescription medications and has not had any problems with concentration or memory. The patient's most likely diagnosis is:
A. Subarachnoid hemmorhage B. Cranial Neuralgia C. Brain tumor D. Posttraumatic headache E. Transient ischemic attack (stroke)
304) A male patient in his thirties presents to you, the family physician, with a chief complaint of fatigue and weakness. He notices he feels tired and becomes short of breath when he engages in everyday tasks that were previously not strenuous for him since about four weeks ago. He states that his condition has neither worsened nor improved since the time he began noticing these symptoms. He also states that he experiences no other associated symptoms. You take a history and find that he is otherwise healthy, but he admits he is a heavy drinker. You proceed with a physical exam and find that he is experiencing tachycardia (he just walked up a flight of stairs to your office). You also find that his muscle strength is normal. What is the most likely diagnosis?
A. Lambert-Eaten syndrome B. Anemia C. Myasthenia Gravis D. Chronic Fatigue Syndrome E. Major Depression
305) 33 year old male patient in the Navy Reserves requires a physical. Upon palpation of the supraclavicular nodes you notice that the left nodes feel normal, but the right nodes are enlarged and more firm. One of the right nodes measures 3.5cm in diameter. Upon further questioning the man claims that he works on a farm when he is not drilling. Due to his day job he you notice he has some cuts in his hands and some scratches on his neck. The next step could be to do what?
A. Send the patient home with an antibiotic prescription. B. Run an immediate biopsy. C. Tell the patient to come back in 1-2 weeks to see if the nodes are still enlarged D. Have a blood culture done E. Administer a vaccine.
305) A 26 year old woman comes to the ER and complaints of severe right-sided chest pain with radiation to the right periscapular region and it is distinctly exacerbated with breathing and movement. She denies any left-sided discomfort and has some dyspnea. Her air saturation is 98%. Cardiovascular survey and abdomen is benign. What is the most primary diagnosis
A) Exacerbated COPD B) Acute Right Pneumothorax C) Asthma D) Pericarditis
306) A 72 year old male presents with ataxia, vertigo and impaired pain and thermal sense over half of the body. You notice he also shows signs of horner's syndrome. You determine that he has had a stroke. Where is the stroke located?
a) Middle cerebral artery b) Posterior cerebral artery c)Basilar artery d) Vertebral artery
307) While performing deep tendon reflexes during a neurological exam, you find that the Right biceps and supinator reflex are +2. the Left bicep and supinator reflex are +1. which spinal segment (s) are/is most likely the cause of this finding.
A) Right cervical 5,6 B) Left cervical 8 C) Right cervical 6,7 D) Left cervical 5,6
308) A 37 yr old female presents to your primary care office complaining of being "always tired and cold". After running lab tests, you find the patient has decreased serum T4. What is the most common way of treating this?
a)50 micrograms levothyroxine once a day b)125 micrograms levothyroxine once a day c)50 mg every 6 hours d)carbimazole 10mg bid
309) You are performing a neurologic exam on a 35 year old female patient. During the exam you notice absent contraction of temporal and masseter muscles on ONE side. Which of the following statements is MOST true regarding her condition? A) Lesion of CN III B) Lesion of CN VII C) Lesion of CN V D) Lesion of CN VI E) Lesion of CN IV
310) Upon presentation to your office, you begin an arterial examination on a middle aged man. The femoral pulse appears to be normal, but the popliteal pulse is decreased or absent. What is the most likely cause of the patients situation?
A.) Aneurysm B.) Superficial thrombophlebitis C.) Venous insufficency D.) Arteriosclerosis Obliterans E.) Lymphadenopathy
311) A semi-conscious patient is brought into the emergency department of the hospital. His speech is extremely slurred and he cannot walk or stand on his own. His physical exam and lab results reveal a number of problems including liver disease, pancreatitis, gastritis, and peripheral neuropathy. Which of the following symptoms/disorders is MOST likely also present?
A. Botulism B. Eosinophilia C. Cystic fibromyalgia D. Acute nitrous oxide intoxication E. Charcot-Marie-Tomkowiak syndrome F. Vitamin B deficiency G. Viral hepatitis H. Eczema I. Yellow fever
312) An adult male arrives at the ER. Your first observation is that he is breathing abnormally. The abnormal breathing pattern can be described as alternating between periods of hyperpnea and periods of apnea. Based only on this first observation, what is the least likely cause of the abnormal breathing?
A) Heart failure B) Asthma C) Uremia D) Drug-induced respiratory depression E) Brain damage
313) A 55 year old male patient comes to the Emergency room complaining of a squeezing pain in the chest that is radiating down the left arm. Upon examination of the patient, the physician finds that patient is experience S4. What is the likely diagnosis.
A. Acute Myocardial Infarction B. Aortic dissection C. Acute Pericarditis D. Pulmonary Embolism E. Acute Pneumothorax
314) A soccer player stops suddenly during a game and feels a pop in her knee. When she presents to your office, you have her lie down in the supine position and flex her knee to 30 degrees. You then stabilize the distal femur with one hand, grasp the proximal tibia in the other hand, and attempt to sublux the tibia anteriorly. What test are you doing and what ligament are you testing?
A. Anterior Drawer Test, ACL B. Posterior Drawer Test, PCL C. Lachman Test, ACL D. Lachman Test, PCL E. Anterior Drawer Test, PCL
315) Patient comes in to the hospital complaining about the change in the level of consciousness and describes his symptoms as having a "State of which vigorous stimuli are needed to elicit a response". Which of the following refers to his complaint? 1. Coma 2. Stupor 3. Confusion 4. Complication 5. Stroke
316) Pt is a 65 year old male. Pt presents with a high fever, chills, dyspnea, and chest pain. Pt produces a purelent, blood-streaked sputum. While taking patient's history you discover that pt recently had an acute upper respiratory infection. Pt is also a non-smoker. Pt problem may be associated with:
a.) Pulmonary Tuberculosis b.) Chronic Bronchitis c.) Cancer of the lung d.) Bacterial Pneumonia e.) Viral Pneumonia
317) A new patient arrives at your office that you have never seen before. When asking questions about his history he has a tendency to digress from one topic to another and then to another never really finishing his line of thought. This gentlemen is most likely exhibiting what type of though process abnormality?
1- Flight of Ideas 2- Echolalia 3- Neologism 4- Tangential 5- Loose Association
318) While speaking with a patient diagnosed with schizophrenia, you notice the patient keeps repeating the same word over and over again, this patient is exhibiting what? a. confabulation b. incoherence c. echolalia d. clanging
319) 48 y.o. male comes in with steady left lower quadrant pain. Patient aslo presents fever and has been constipating for few days. Which of the following he is likely to have:
A. Bilary Colic B. Acute Cholecystitis C. Acute diverticulitis D. Acute, appendicitis E. Chronic Pancreatitis
320) While on an externship, you are asked to obtain a complete history from a 45 year old male. At the beginning of the interview you observe the patient has a neatly trimmed beard on the left side of his face, while the right side of his beard seems ungroomed. He tells you he is left handed. What might this indicate?
A. the patient has a lesion in his left parietal cortex B. the patient has a lesion in his right parietal cortex C. the patient has a lesion in his left temporal cortex D. the patient has a lesion in his right temporal cortex E. the patient has a lesion in his left frontal cortex
321) A man who has just recently suffered from a stroke has chronic contralateral weakness and spasticity. When testing his reflexes you find weakness when he extends his right arm and when he dosiflexes is right foot. You also notice that his leg is externally rotated at his hip. He complains of sensory loss in both his right arm and his right foot and you observe increased tendon reflexes. Based on these clinical observations you suspect the location of the lesion to be in:
A) Left Cerebral Cortex B) Right Cerebral Cortex C) Brainstem D) Spinal Cord above T1 E) Cerebellum
322) A 55 year old female comes in for her annual physical examination and complains of abdominal pain in the epigastric region. She relates the pain has recently become an issue and has slowly progressed. Food often aggravates her condition. Antacids do not provide relief. Physical exam reveals significant weight loss. What is the most likely diagnosis?
a) chronic pancreatitis b) peptic ulcer c) acute appendicitis d) cancer of the stomach e) cancer of the intestines
323) A patient comes into the office complaining of wheezing, dyspnea, cough, and shortness of breath. Based on the symptoms presented, what could the patient be suffering from?
A. Obstructive Pulmonary Diseases B. Asthma C. Lung Cancer D. Myocardial Infarction E. Whooping Cough
324) A positive anterior drawer sign suggests a tear of: A) Medial collateral ligament (MCL) B) Lateral collateral ligament (LCL) C) Anterior cruciate ligament (ACL) D) Posterior cruciate ligament (PCL)
325) A 66 year old male comes into the emergency room complaining of fever, dyspnea, and coughing up blood. While taking his history, you discover he is diabetic, and has been feeling weak with a headache and fever and chill over the last week. You order a chest x-ray and discover diffuse infiltrates. The most probable diagnosis would be what? A) SARS B) Common cold C) Complicated Influenza D) Bronchiolitis
E) Laryngotracheobronchitis
326) Which one of the following CSF analysis findings is abnormal? A) clear, colorless and odorless appearance B) minimal WBC and no RBC C) Positive for glucose and protein D) pH 9
327) A fifty year old man is brought into the emergency room by his daughter. She tells you that the last time she saw her father was five days ago when she gave him an ultimatum, stop drinking or you can longer be a part of my life. She says he showed up at her apartment today shaking, sweating, breathing heavily, and yelling at her about a ghost invasion. You determine that her father is suffering from severe alcohol withdrawal, what is the first treatment you administer this patient?
a. Multiple B vitamins including thiamine b. Methadone bid c. High dose Benzodiazepines d. Haloperidol and Fluphenazol e. Naltrexone
328) A patient comes in complaining of a sudden onset of a terrible headache. He's complaining of numbness and tingling on his left side, particularly his left arm. His left leg is not affected. You also notice that his eyes deviate to the right side and that his speech is slow, as he has difficulty finding the right words to say. Which of the following arteries is most likely affected?
a) right anterior cerebral a.
b) left middle cerebral a. c) right middle cerebral a. d) posterior cerebral a. e) basilar a.
329) A 40y/o patient comes into the office complaining of persistent pain over her abdomin and back with an acute onset. She says it is worse when lying down on her back to sleep but better if she bends over. She complains of nausea, vomiting, abdominal distention, and fever. What is the primary diagnosis for this patient?
a) biliary colic b) chronic pancreatitis c) acute pancreatitis d) actute appendicitis e) acute mechanical intestinal obstruction
330) A 32 year old man presents to you with steady abdominal pain. The patient also claims he has nausea, vomiting, and fever. The pain seems to stay in the epigastric region but also radiates to the back and other parts of the abdomen. The patient claimed the pain would become less aggravating and bearable when he leaned forward.
A) Dyspepsia B) Acute Diverticulitis C) Acute Appendicitis D) Acute Pancreatitis
331) During a physical exam your patient is unable to balance on his own. Your patient is not able to stand with his eyes closed without losing his balance. What might be the cause of this abnormality?
A. lower motor lesion B. upper motor lesion C. Middle motor lesion D. cerebellar disfunction E. dysphagia
332) A 65 year male comes to your office complaining of discoloration and pain in his legs. You notice there is slight cyanosis accompanied with hyperpigmentation and edema. Which of the following is part of your diagnosis?
a) Raynaud's Phenomenon b) Venous stasis ulcers c) Superficial thrombophlebitis d) Cellulitis e) Erythema nodosum
333) A 52 year old male comes to your office complaining of "Back Pain." Which type of pain is this patient now experiencing? A. Chronic Progressive Pain B. Chronic Periodic Pain C. Chronic Intractable Pain D. Acute Pain E. None of the Above
334) A 39 year old female, accompanied by her husband, has come into the ER with pain in the lower right quadrant of her abdomen. As she holds her husband's hand in anguish, the woman relates that the pain began at least 5-6 hours earlier and has progressed to an unbearable pain. Because of the nature of this history and patient pain what is the first thing that should be done? A. Give the patient medication to manage the pain until the operating room opens. B. Quickly asses the lower left quadrant of the abdomen for signs of appendix rupture. C. You need to get patient's permission for husband to stay before potentially lifesaving diagnostic findings can be assessed. D. After quickly making the diagnosis of a ruptured appendix, promptly go to surgery. E. Osculate abdomen before any palpations are made.
335) Which disease, characterized by altered bowel habits, abdominal pain, and absence of detectable organic pathology, is the most common GI disease in clinical practice? A. Cholelithiasis B. Cholecystitis C. Irritable Bowel Syndrome D. Peptic Ulcer Disease E. Crohn's Disease
336) Which of the following is considered a definition of CAGE?
A.Crazy B.Assault C.Guilty D.Expert E.Cautious
337) While performing the gait portion of a physical exam on a patient, you notice that the patient walks as though they are going up stairs while walking across the exam room. You then ask the patient to attempt to walk on his heels across the room, but he is unable to perform this task. What is the most likely cause of this gait abnormality? A. Spastic paresis B. Lower Motor Neuron lesion C. Upper Motor Neuron lesion D. Corticospinal tract lesion
338) A patient comes in complaining of pain that begin around the umbilicus and is now in the right lower quadrant of their abdomen. You believe think they may have appendicitis and begin to examen them. The patient has a positive obturator sign. What maneuver is performed to produce a positive obturator sign?
A. Press deeply and evenly in the left lower quadrant. B. Ask the patient to raise their thigh against your hand. C. Flex the patient's right thigh at the hip and rotate the leg internally at the hip. D. Gently pick up a fold of skin between your thumb and index finger. E. Hook your left thumb under the costal margin at the point where the lateral border of the rectus muscle intersects the costal margin.
339) A 60 yr. old male admits to the E. R. with chest pain, which he relates as sharp and 7 out of 10. He admits to a 15 pack year smoking history. He describes palpatations and shortness of breath with the chest pain. No edema is present. His BP is 135/98 Without further testing, what is your primary diagnosis?
A. Angina pectoris B. GERD C. Myocardial infarction D. Congestive heart failure E. Aortic dissection
340) A patient with urosepsis presents drowsy and falls asleep several times during the physical examination. The patient is easily aroused, is cooperative and responds to questions and commands appropriately. Which word best describes the woman? A. Lethargic B. Stuporous C. Comatose D. Obtunded E. Alert
341) A 23 y/o female presents to the clinic complaining of fatigue, dry hair and skin, cold intolerance, and mild weight gain with a decrease in appetitie. Upon examination you discover a presence of bradycardia, hypertension, and cool peripheral extremities. Lab results indicate a decrease in serum T4. The most likely diagnosis is:
A. anemia B. hypothyroidism C. dehydration D. hyperthyroidism E. growth hormone deficiency.
342) A 45 year old male presents to your office complaining of episodic burning pain in his esophagus. He relates that this burning sensation usually occurs an hour and a half to three hours after eating large meals. He states that his symptoms are worse when he eats close to his bed time and are partially alleviated by eating more food. What is the most likely cause of this clinical presentation?
A. Duodenal Ulcer B. Gastric Ulcer C. Aortic Aneurysm D. Asthma E. Tumor
343) a 17 year old boy has a 1-day history of pain in the left ear. He is a member of his high school swimming team. The left ear canal is swollen, erythematous, and tender. He has pain when the auricle is pulled. Which of the following is the most likely diagnosis?
a) Acute otitis media b) Chronic otitis media c) Acute otitis externa d) Lyme's disease e) None of the above
344) Which of the following is not one of the seven attributes of a symptom?
a) Location b) Effect of problem on patient's life c) Associated manifestations/symptoms d) Timing e) Quality
345) A 56 year old female presents with anterior chest and mid-back pain that she describes as "tearing" or "ripping". She has a history of hypertension. You examine her and you find weak asymmetric peripheral pulses, and a diastolic murmur. You take a Chest X-rays which reveals a widened mediastinal silhouette. What condition is this patient most likley suffering from:
A. Acute Myocardial Infarction B. Aortic Dissection C. Acute Pericarditis D. Pulmonary Embolism E. Acute Pneumothorax
346) A 24 yr old male presents to your office complaining of recurrent headaches at night for the past 7 weeks. The patient describes the pain as deep or searing around and behind his left eye. He has noticed that his eye often waters and has complained of nasal congestion. He has several headaches each night and notices that they occur around the same time. He denies nausea, vomiting, and visual abnormalities. He tells you that alcohol sometimes seems to provoke the headaches. What is the most likely diagnosis?
A) Tension Headache B) Migraine without aura C) Cluster Headache D) Brain Tumor E) Headache from Sinusitis
347) A 19 y.o. patient is referred to you with a suspected speech problem. You listen to her spontaneous speech and notice that it is very slow and takes a great deal of effort for her to speak a few words, but those words are meaningful with nouns, transitive verbs, and important adjectives. Her inflection and articulation are also impaired. You suggest it is Broca's Aphasia. Where on the brain would you suspect a lesion to be located?
A. Posterior superior temporal lobe B. Posterior inferior temporal lobe C. Occipital lobe D. Posterior superior frontal lobe E. Posterior inferior frontal lobe
348) Pt. comes to clinic with complaints of sleepless nights, irritability, palpitations, and excessive sweating. Upon further testing you diagnosis Grave's Disease. Which of the following treatments is contraindicated in pregnancy?
A. Methimazole B. Carbimazole C. Propylthiouracil (PTU) D. Radioiodine E. Propranolol
349) When palpating the thyroid on a patient during a physical exam, you notice that the thyroid is soft when palpating it. What is this an indication of?
A. Hyperthroidism B. Hashimoto's thyroiditis C. Graves' Disease D. Hypothyroidism
350) 52-year old patient comes into your office complaining of migraines, back pain that won't go away, and a pain in his knee from having banged it on your office door as he came into the examining room. Of the following types of pain, which does he NOT display?
a) chronic intractable b) chronic progressive c) acute d) chronic periodic e) both a & b
351) What is the proper order for an abdominal exam?
A. palpate, percuss, auscultate, inspect B. inspect percuss, palpate, auscultate C. inspect, auscultate, percuss, palpate d. auscultate, inspect, percuss, palpate
352) A 43 yr old male comes into your office complaining of midepigastric pain that radiates to his back and becomes worse when he lays down. Upon examining the Pt. you note diminished bowel sounds and pleural effusion. What would be the best test to confirm your diagnosis.
A. Abdominal radiograph B. Ultrasound C.chest x-ray D. CT E. Sinogram
353) A 35 year old male patient comes into your office. He is anxious and uncomfortable. You decide to assess his mental health and upon questioning, learn that he fears criticism and avoids interpersonal relationships. He does not exhibit antisocial behavior and he does not seem paranoid. You classify him as:
a. Axis I, cluster A b. Axis I, cluster C c. Axis II, cluster A d. Axis II, cluster B e. Axis II, cluster C
354) A 29 year old female presents to your clinic complaining of symptoms of lethargy, dry skin, bradycardia, mild diastolic hypertension, and cool peripheral extremities. What is the most likely cause of her symptoms?
Hyperthyroidism Graves disease Hypothyroidism Alcoholism Diabetes Insipidus
355) A 75 year old woman comes to your clinic complaining of fever, cough and difficulty breathing. On physical examination you note tachypnea, increased tactile and vocal fremitus, egophony, whispered pectoriloquy, dullness to percussion, and a respiratory rate of 33/min. What is the most likely diagnosis?
a.) Lung Cancer b.) Pneumonia c.) Asthma d.) Pleural Effusion e.) Pneumothorax
356) A 45 year old woman with Marfan Syndrome, hyperlipidemia, and history of hypertension is brought to the hospital after feeling a "ripping" sensation throughout the chest and neck. She claims that the location of the pain has traveled since she first felt the ripping quality. Upon examination she reveals sinus tachycardia and hypotension. What is the most likely diagnoses?
A. Hypovolemic Shock B. Aortic Dissection C. Thoracic Outlet Syndrome D. Aortic Aneurysm E. Myocardial Infarction
357) A 23 year old male presents to the clinic complaining of headaches that have been occurring almost nightly for the past 4 weeks. The headaches usually occur while he is sleeping and last for about an hour. The patient denies any nausea or vomiting associated with the headaches. The patient states that he had similar episodic headaches on a couple of occasions over the past couple years. Upon review of the patients social history you discover that he consumes alcohol several times a week. What is your most likely working diagnosis?
A) Post-concussion headache B) Cluster Headache C) Common Migraine D) Tension Headache
358) Mr. Stephanopolous comes to your office for a routine blood check. The results come back, and you realize he has an increased hematocrit and RBC mass. His serum EPO levels are elevated. Mr. Stephanoplous doesn't somke and his oxygen saturation is 100%. His hemoglobin oxygen affinity is also normal. Which one of the following could be a potential diagnosis?
A. Smoker's Polycthemia B. Polycythemia vera C. Uternia leiomyoma D. Macrocytic anemia E. Relative Erythrocytosis
359) Upon examination of a 48 yr old female with family history of mental illness you find the patient to have a "Misinterpretation of real external stimuli" Which of the following abnormalities of perception could describe the patients condition?
A. Delusions B. Echolalia C. Hallucinations D. Illusions E. Blocking
360) A 26 year old female patient presents to you complaining of recently feeling "on edge" lately in social situations. The patient states that she has been excessively worrying about minor problems that in the past she would not be concerned about. She also has been suffering episodes of muscle tension and insomnia. The patient denies having shortness of breath, palpitations, or tachycardia. What psychiatric disorder is your patient most likely suffering from?
A. Panic Disorder B. Generalized Anxiety Disorder C. Obsessive-Compulsive Disorder D. Posttraumatic Stress Disorder E. Schizophrenia
361) Mary comes into your office. She is wearing a hot-pink sneaker on her right foot and a yellow flip-flop on her left foot. She is talking with a normal tone, loudness and rate about how she is going skydiving this afternoon and needs to wrap up your session early so she can take a bus from which she will parachute from.
Which aspects of the mental status exam do you gain insight to in the above anecdote?
A. Appearance/Behavior & Affect B. Concentration and Mood C. Appearance/Behavior & Concentration D. Appearance/Behavior & Thought Process/Content E. Appearance/Behavior & Speech
362) A patient is brought into your clinic by a concerned relative. The relative states that the patient has been constantly tired and not at all alert. When you try to speak to the patient, he vaguely responds. You decide to do a mental status exam. You take note that the patient is drowsy, but keeps his eyes open as he looks at you. He does respond to questions, and does NOT seem confused by them. However, he sporadically falls asleep, and you must continuously wake him. This patient can be described as:
A. Obtuned B. Lethargic C. Anxious D. Euphoric E. Aphasic
363) A 45 yr old man presents to the clinic with tenderness over the plantar surface of his left foot around the area of his third and fourth metatarsal heads. He also complains of hyperesthesia, numbness, aching, and burning from the metatarsal heads into the third and fourth toes. The most likely diagnosis for this patient would be.
A Rheumatoid arthritis B plantar fasciitis C metatarsalgia due to vascular compromise D tarsal tunnel syndrome E Morton's neuroma
364) A 79 year old female patient presents to your office complaining of headache, fever, chills, myalgia and malaise. The patient is diabetic and has also developed cough and sore throat. A throat swab is performed and on further evaluation you diagnose this patient as having influenza. What is the most significant complication that this patient is at risk of contriving?
A. Asthma B. Bronchitis C. Pneumonia. D. Tuberculosis.
365) A 55 year old female patient is brought to the emergency room as a result of a multitude of symptoms. Upon physical examination you immediately notice that her eyes are gazing to the left, and she is is unable to move one side of her body. Furthermore, although conscious, her speech is very hard to comprehend. If you diagnose this patient with a stroke, what artery is likely the cause of these symptoms?
a) Right Middle Cerebral Artery b) Left Posterior Cerebral Artery c) Left Anterior Cerebral Artery d) Left Middle Cerebral Artery e) None of the above
366) A 45 y.o. male presents to the clinic complaining of lower back pain. He relates that the pain begain yesterday while helping his daughter move into the dorms. While trying to lift her futon he felt severe pain in his lower back. During your physical exam you notice the achilles reflex is absent and there is diminished sensation around his lateral malleolus. Suspecting this is a heriated disc, what nerve root is affected?
A. L4 B. L5 C.S1 D. S2
367) A 44 year old male cigarette smoker presents to you complaining of regular calf cramps after walking short distances. Patient relates to pain being relieved with rest and aggravated with standing for long periods of time or with exercise. Leg cramps are also present at night, often more painful episodes than during the day. Which of the following is the best diagnosis for this patient?
A. DVT B. Thromboangiitis Obliterans C. Raynaud's Disease D. Thrombophlebitis E. Erythema Nodosum
368) A 35 year old female visits her family physician. She complains of anxiety, palpitations, intolerance to heat, nervousness with trembling hands, as well as weight loss. She has a normal appetite. On Physical Exam she presents with tachycardia, a BP of 150/80, sweaty palms, exophthalmos and a generalized enlargement of the thyroid gland. You decide to perform lab tests. Her lab results indicate that she has a decreased TSH level as well as an increased T3, T4, and also free T4. Based upon all of the above information you conclude that she has:
A. Anxiety Disorder B. Hyperthyroidism C. Anemia D. Hypothyroidism E. Hydrocephalus
369) 35 yo pregnant female presents to the clinic with excsesive sweating, nervousness, and severe weight loss. She complains that her fingernails are separating from her nail bed. Upon phyiscal exam you notice her thyroid is enlarged 3 times its normal size. What is your treatment?
A. Carbimazole B. Propranolol C. radioiodine D. Propylthiouracil (PTU) E. Warfarin
370) A 28 yr old women presents to her physician experiencing of feeling a hyperalertness whenever she hears a loud noise. Her husband tells the physician she has been avoiding the outside world lately. The lady is an army veteran. What would be your best diagnosis.
A. Panic Disorder B. Depression C. Bipolar Disorder D. Schizophrenia E. Posttraumatic Stress Disorder
371) A 45 years old man presents to the ER with high fever, headache, non-productive cough and severe dyspnea. Three days ago, he experienced abrubt onset of chills, diarrhea, fever, and headache after he came back from a trip to visit his sick mother at the hospital in Boston. He also complains of his chest pain. Upon palpation and examination, his respiratory rate was 28/min. He is shown dullness percussion on his chest, and egophony is heard. No major medical history on the record. He has no allergies on medication. What is the most likely diagnosis?
a) interstitial nephritis b) asthma and hypersensitivity pneumonitis c) Pneumonia d) Enterovirus e) gestrointestinal bleeding
372) A patient comes into your office complaining of right ankle and foot weakness. The patient relates the muscle weakness has caused him "trip and fall" several times in the past couple of weeks. As part of your physical exam, you note sensory loss on the dorsal surface of the right foot and lateral side of the right lower leg. You also note, ankle joint dorsiflexion and subtalar joint eversion is decreased for the right foot. As part of your differential diagnosis, you include lumbosacral radiculopathy. Which lumbosacral nerve root would most likely be affected?
A. L2 B. L3 C. L4 D. L5 E. S1
373) What is the most common clinical manifestation of coronary artery disease? 1.chronic atrial fibrillation 2. angina pectoris 3. myocardial infarction 4. hypertrophic obstructive cardiomyopathy 5.mitral stenosis
374) A patient presents to you complaining of ankle pain and instability. The patient relates to "twisting" his right ankle yesterday while playing basketball. Upon examination the right ankle is swollen and tender on the anterolateral aspect, and the left ankle is unremarkable. You determine the cause of injury was an inversion injury. Which of the following ligaments is most at risk for an inversion injury?
A. Calcaneofibular ligament B. Anterior talofibular ligament C. Posterior talofibular ligament D. Tibionavicular ligament E. Anterior talotibial ligament
375) Which of the following drugs to treat alcoholism would cause the irreversible inhibition of aldehyde dehydrogenase leading to a patient having a severe or life-threatening reaction to alcohol?
a. naltrexone b. disulfiram c. acamprosate d. fomepizole e. naloxone
376) A 28-year-old woman seeks advice from her family physician after experiencing several episodes of discomfort. The most recent event consisted of symptoms of severe discomfort while watching her favorite baseball team play to a full crowd at their home stadium. Her symptoms included difficulty breathing, dizziness and rapid heart rate. Although her episodes do not last very long, she has stopped attending games and avoids social situations that provoke an episode. The patients symptoms would most likely be explained by which of the following disorders?
a.Social Phobia b.Obsessive-Compulsive Disorder c.Specific Phobia d.Panic Disorder e.Posttraumatic Stress Disorder